Module 6 - Chapter 30: Management of Patients with Hematologic Neoplasms

¡Supera tus tareas y exámenes ahora con Quizwiz!

The nurse is caring for a patient with acute myeloid leukemia (AML) with high uric acid levels. What medication does the nurse anticipate administering that will prevent crystallization of uric acid and stone formation?

Allopurinol (Zyloprim) Massive leukemic cell destruction from chemotherapy results in the release of intracellular electrolytes and fluids into the systemic circulation. Increases in uric acid levels, potassium, and phosphate are seen; this process is referred to as tumor lysis (cell destruction) syndrome. The increased uric acid and phosphorus levels make the patient vulnerable to renal stone formation and renal colic, which can progress to acute renal failure. Patients require a high fluid intake, and prophylaxis with allopurinol (Zyloprim) to prevent crystallization of uric acid and subsequent stone formation.

A nurse has established for a client the nursing diagnosis of risk for infection. Which of the following interventions would the nurse include in the plan of care for this client? Select all answers that apply. - Assess skin and mucus membranes every shift. - Provide oral hygiene once daily. - Encourage the client to take deep breaths every 4 hours while awake. - Place fresh flowers on a shelf on the opposite wall from the client. - Auscultate lung sounds every shift and prn.

- Assess skin and mucus membranes every shift. - Encourage the client to take deep breaths every 4 hours while awake. - Auscultate lung sounds every shift and prn.

The nurse is caring for a client with acute myeloid leukemia (AML) with high uric acid levels. What medication does the nurse anticipate administering that will prevent crystallization of uric acid and stone formation? Allopurinol Filgrastim Hydroxyurea Asparaginase

Allopurinol Explanation: Massive leukemic cell destruction from chemotherapy results in the release of intracellular electrolytes and fluids into the systemic circulation. Increases in uric acid levels, potassium, and phosphate are seen; this process is referred to as tumor lysis (cell destruction) syndrome. The increased uric acid and phosphorus levels make the client vulnerable to renal stone formation and renal colic, which can progress to acute renal failure. Clients require a high fluid intake, and prophylaxis with allopurinol (Zyloprim) to prevent crystallization of uric acid and subsequent stone formation.

A nurse is assessing a client with multiple myeloma. Due to this condition, what will this client be at risk for? A.) chronic liver failure. B.) acute heart failure. C.) pathologic bone fractures. D.) hypoxemia.

Answer: C.) pathologic bone fractures.

A client has completed induction therapy and has diarrhea and severe mucositis. What is the appropriate nursing goal? A.) Address issues of negative body image. B.) Place the client in reverse isolation. C.) Administer pain medication. D.) Maintain nutrition.

Answer: D.) Maintain nutrition.

A client is receiving chemotherapy for acute myeloid leukemia and has poor nutritional intake. What is the first action the nurse should take?

Ask, "Are you experiencing nausea?"

A client presents with peripheral neuropathy and hypothesia of the feet. What is the best nursing intervention? Elevate the client's legs. Encourage ambulation. Assess for signs of injury. Keep the feet cool.

Assess for signs of injury. Explanation: A client with hypothesia of the feet will have decreased sensation and numbness. The nurse should assess for signs of injury. If the client is injured, he or she will not be able to feel it; this could lead to the development of infection. Ambulation will not help the client, and elevating the legs may make the problem worse, as blood flow to the feet would be decreased. Keeping the feet cold will also decrease blood flow.

A client who is undergoing chemotherapy for AML reports pain in the low back. What is the nurse's first action?

Assess renal function

A client who is undergoing chemotherapy for AML reports pain in the low back. What is the nurse's first action?

Assess renal function.

A client with AML has pale mucous membranes and bruises on the legs. What is the primarynursing intervention? Assess the client's skin. Assess the client's hemoglobin and platelets. Assess the client's pulse and blood pressure. Check the client's history.

Assess the client's hemoglobin and platelets. Explanation: Clients with AML may develop pallor from anemia and a tendency to bleed because of a low platelet count. Assessing the client's hemoglobin and platelets will help to determine whether this is the cause of the symptoms. This would be the priority above assessing pulses, blood pressure, history, or skin.

The nurse is caring for a client with chronic myeloid leukemia (CML) who is taking imatinib mesylate. In what phase of the leukemia does the nurse understand that this medication is most useful to induce remission?

Chronic

A client with multiple myeloma is complaining about pain. What instructions will the nurse give the client to help to reduce pain during activity? Do not lift more than 10 pounds. Stay in bed as much as possible. Limit activity to once a day. Limit fluids to prevent going to the bathroom.

Do not lift more than 10 pounds. Explanation: The client with multiple myeloma needs education about activity instructions, such as lifting no more than 10 pounds and using proper body mechanics. Braces may be needed. The clent should be active and would not be instructed to stay in bed or limit activity, as he or she would become very stiff. Limiting fluids would be contraindicated; the client needs to remain well hydrated.

Which precautions should a nurse include in the care plan for a client with leukemia and neutropenia?

Eliminate fresh fruits and vegetables, avoid using enemas, and practice frequent hand washing

T or F- Polycythemia vera is a condition that places the patient at risk for increased infection.

False

When assessing a female pt with a disorder of the hematopoietic or the lymphatic system, which of the following assessments is most essential?

Health hx, such as bleeding, fatigue, and fainting

A nurse is caring for a client with multiple myeloma. What lab value is the nurse most likely to see?

Hypercalcemia

A client has been diagnosed with multiple myeloma. Which of the following laboratory values should the nurse expect to find in a client with multiple myeloma?

Increased urinary protein A characteristic finding in multiple myeloma is protein in the urine. Other laboratory findings include increased serum protein, hypercalcemia, anemia, and hyperuricemia. Polycythemia vera is not found in multiple myeloma.

A patient with AML is having aggressive chemotherapy to attempt to achieve remission. The patient is aware that hospitalization will be necessary for several weeks. What type of therapy will the nurse explain that the patient will receive?

Induction Therapy

A patient with AML is having aggressive chemotherapy to attempt to achieve remission. The patient is aware that hospitalization will be necessary for several weeks. What type of therapy will the nurse explain that the patient will receive?

Induction therapy

The nurse is administering packed red blood cell (RBC) transfusions for a patient with myelodysplastic syndrome (MDS). The patient has had several transfusions and is likely to receive several more. What is a priority for the nurse to monitor related to the transfusions?

Iron levels

A nurse caring for a client with myeloma prepares to administer dexamethasone to the client. What is the nurse's best understanding of how this medication is an effective treatment option for this client? It kills affected cells. It decreases immune response. It decreases tumor necrosis factor. It kills affected bone marrow.

It kills affected cells. Explanation: Dexamethasone is used to induce myeloma apoptosis and cell death and to reduce bone pain.

A client has completed induction therapy and has diarrhea and severe mucositis. What is the appropriate nursing goal?

Maintain nutrition. Maintaining nutrition is the most important goal after induction therapy because the client experiences severe diarrhea and can easily become nutritionally deficient and develop fluid and electrolyte imbalance. The client is most likely not in pain at this point, and this is an intervention, not a goal.

A client has completed induction therapy and has diarrhea and severe mucositis. What is the appropriate nursing goal? - Address issues of negative body image. - Administer pain medication. - Place the client in reverse isolation. - Maintain nutrition.

Maintain nutrition. Maintaining nutrition is the most important goal after induction therapy because the client experiences severe diarrhea and can easily become nutritionally deficient and develop fluid and electrolyte imbalance. The client is most likely not in pain at this point, and this is an intervention, not a goal.

A client was admitted to the hospital with a pathologic pelvic fracture. The client informs the nurse that he has been having a strange pain in the pelvic area for a couple of weeks that was getting worse with activity prior to the fracture. What does the nurse suspect may be occurring based on these symptoms? Hemolytic anemia Polycythemia vera Leukemia Multiple myeloma

Multiple myeloma Explanation: The first symptom usually is vague pain in the pelvis, spine, or ribs. As the disease progresses, the pain becomes more severe and localized. The pain intensifies with activity and is relieved by rest. When tumors replace bone marrow, pathologic fractures develop. Hemolytic anemia does not result in pathologic fractures nor does polycythemia vera or leukemia.

Which cell of the haematopoiesis is responsible for the production of RBCs and platelets?

Myeloid stem cells

Which term refers to an abnormal decrease in white blood cells, red blood cells, and platelets? Pancytopenia Anemia Leukopenia Thrombocytopenia

Pancytopenia Explanation: Pancytopenia may be congenital or acquired. Anemia refers to decreased red cell mass. Leukopenia refers to a less-than-normal amount of WBCs in circulation. Thrombocytopenia refers to a lower-than-normal platelet count.

A patient with polycythemia vera has a high red blood cell (RBC) count and is at risk for the development of thrombosis. What treatment is important to reduce blood viscosity and to deplete the patient's iron stores?

Phlebotomy The objective of management is to reduce the high RBC count and reduce the risk of thrombosis. Phlebotomy is an important part of therapy (Fig. 34-5). It involves removing enough blood (initially 500 mL once or twice weekly) to reduce blood viscosity and to deplete the patient's iron stores, thereby rendering the patient iron deficient and consequently unable to continue to manufacture hemoglobin excessively.

The nurse is performing an assessment on a patient with acute myeloid leukemia (AML) and observes multiple areas of ecchymosis and petechiae. What laboratory study should the nurse be concerned about?

Platelet count of 9,000/mm3

The nurse is performing an assessment on a patient with acute myeloid leukemia (AML) and observes multiple areas of ecchymosis and petechiae. What laboratory study should the nurse be concerned about? WBC count of 4,200 cells/mcL Hematocrit of 38% Platelet count of 9,000/mm3 Creatinine level of 1.0 mg/dL

Platelet count of 9,000/mm3 Explanation: Complications of AML include bleeding and infection, which are the major causes of death. The risk of bleeding correlates with the level and duration of platelet deficiency (thrombocytopenia). The low platelet count can cause ecchymoses (bruises) and petechiae. Major hemorrhages also may develop when the platelet count drops to less than 10,000/mm3.

The nurse is performing an assessment on a patient with acute myeloid leukemia (AML) and observes multiple areas of ecchymosis and petechiae. What laboratory study should the nurse be concerned about? - Platelet count of 9,000/mm3 - WBC count of 4,200 cells/mcL - Hematocrit of 38% - Creatinine level of 1.0 mg/dL

Platelet count of 9,000/mm3 Complications of AML include bleeding and infection, which are the major causes of death. The risk of bleeding correlates with the level and duration of platelet deficiency (thrombocytopenia). The low platelet count can cause ecchymoses (bruises) and petechiae. Major hemorrhages also may develop when the platelet count drops to less than 10,000/mm3.

The nursing instructor is discussing disorders of the hematopoietic system with the pre-nursing pathophysiology class. What disease would the instructor list with a primary characteristic of erythrocytosis?

Polycythemia Vera Polycythemia vera is associated with a rapid proliferation of blood cells produced by the bone marrow. In Sickle Cell disease HbS causes RBCs to assume a sickled shape under hypoxic conditions. Aplastic anemia has a deficiency of erythrocytes. Options B, C, and D do not have the characteristics of erythrocytosis.

The nursing instructor is discussing disorders of the hematopoietic system with the pre-nursing pathophysiology class. What disease would the instructor list with a primary characteristic of erythrocytosis?

Polycythemia vera

The nurse recognizes the clinical assessment of a patient with acute myeloid leukemia (AML) includes observing for signs of infection early. What nursing action will most likely help prevent infection?

Practice vigilant handwashing.

The nurse is assessing a patent with polycythemia vera. What skin assessment data would the nurse determine is a normal finding for this patient? Pale skin and mucous membranes Bronze skin tone Ruddy complexion Jaundice skin and sclera

Ruddy complexion Explanation: Polycythemia vera (sometimes called P vera), or primary polycythemia, is a proliferative disorder of the myeloid stem cells. Patients typically have a ruddy complexion and splenomegaly.

The nurse is assisting the client with multiple myeloma to ambulate. What is the most important nursing diagnosis to help prevent fractures in this client? - Safety - Adequate nutrition - Adequate hydration - Increased mobility

Safety

A client with suspected multiple myeloma is reporting back pain. What is the priority nursing action?

Send the client for a spinal x-ray study.

The term that is used to refer to a primitive cell, capable of self-replication and differentiation is which of the following?

Stem cell

Which term is used to refer to a primitive cell that is capable of self-replication and differentiation? - Reticulocyte - Spherocyte - Band cell - Stem cell

Stem cell Stem cells may differentiate into myeloid or lymphoid stem cells. A band cell is a slightly immature neutrophil. A spherocyte is a red blood cell without central pallor. A reticulocyte is a slightly immature red blood cell.

The nurse is assessing several clients. Which client does the nurse determine is most likely to have Hodgkin lymphoma?

The client with enlarged lymph nodes in the neck.

The nurse is assessing several clients. Which client does the nurse determine is most likely to have Hodgkin lymphoma? - The client with painful lymph nodes under the arm. - The client with enlarged lymph nodes in the neck. - The client with painful lymph nodes in the groin. - The client with a painful sore throat.

The client with enlarged lymph nodes in the neck. Lymph node enlargement in Hodgkin lymphoma is not painful. The client with enlarged lymph nodes in the neck is most likely to have Hodgkin lymphoma if the enlarged nodes are painless. Sore throat is not a sign for this disorder.

A client with polycythemia vera reports gouty arthritis symptoms in the toes and fingers. What is the nurse's best understanding of the pathophysiological reason for this symptom?

The dead red blood cells release excess uric acid.

The nurse assesses a patient for late-stage chronic lymphocytic leukemia (CLL) by looking for what?

Thrombocytopenia

Nursing interventions for the patient with acute leukemia should focus on assisting the patient to establish a balance between ___________ and _______ because fatigue and deconditioning are common symptoms of the disease.

activity, rest

A client has been diagnosed with multiple myeloma. Which of the following laboratory values should the nurse expect to find in a client with multiple myeloma? a) Polycythemia vera b) Decreased serum protein c) Increased urinary protein d) Decreased calcium level

b) Decreased serum protein A characteristic finding in multiple myeloma is protein in the urine. Other laboratory findings include increased serum protein, hypercalcemia, anemia, and hyperuricemia. Polycythemia vera is not found in multiple myeloma.

Which of the following is the only curative treatment for chronic myeloid leukemia (CML)? a) Idarubicin b) Imatinib c) Allogeneic stem cell transplant d) Cytarabine

c) Allogeneic stem cell transplant Allogeneic stem cell transplantation remains the only curative treatment for CML. The efficacy of Imatinib as first-line treatment and the treatment-related mortality of stem cell transplant limits use of transplant to patients with high risk or relapsed disease, or in those patients who did not respond to therapy with TKI. Cytarabine and idarubicin are part of induction therapy for acute myeloid leukemia (AML).

Which of the following terms refers to an abnormal decrease in white blood cells, red blood cells, and platelets? a) Thrombocytopenia b) Anemia c) Pancytopenia d) Leukopenia

c) Pancytopenia Explanation: Pancytopenia may be congenital or acquired. Anemia refers to decreased red cell mass. Leukopenia refers to a less-than-normal amount of WBCs in circulation. Thrombocytopenia refers to a lower-than-normal platelet count.

Lymphadenopathy

enlargement of a lymph node or lymph nodes

The nurse is discussing disorders of the hematopoietic system when a client asked about erythrocytosis. What disease will the nurse mention with a primary characteristic of erythrocytosis?

polycythemia vera

Stem cell

primitive cell, capable of self-replication and differentiation into myeloid or lymphoid stem cell

A patient with acute myeloid leukemia (AML) is having hematopoietic stem cell transplantation (HSCT) with radiation therapy. In which complication do the donor's lymphocytes recognize the patient's body as foreign and set up reactions to attack the foreign host? A.) Acute respiratory distress syndrome B.) Graft-versus-host disease C.) Remission D.) Bone marrow depression

Answer: B.) Graft-versus-host disease

A client was admitted to the hospital with a pathologic pelvic fracture. The client informs the nurse that he has been having a strange pain in the pelvic area for a couple of weeks that was getting worse with activity prior to the fracture. What does the nurse suspect may be occurring based on these symptoms? A.) Hemolytic anemia B.) Polycythemia vera C.) Leukemia D.) Multiple myeloma

Answer: D.) Multiple myeloma

A client who is undergoing chemotherapy for AML reports pain in the low back. What is the nurse's first action?

Assess renal function. Chemotherapy results in the destruction of cells and tumor lysis syndrome. Uric acid and phosphorus concentrations increase, and the client is susceptible to renal failure. The nurse should assess renal function if the client complains of low-back pain, as this could be indicative of kidney stone formation. Heating pads, pain medication, and referrals could be instituted once the cause of the pain is determined. The priority is further assessment to rule out important problems.

A client with AML has pale mucous membranes and bruises on the legs. What is the primary nursing intervention?

Assess the client's hemoglobin and platelets.

A client is undergoing tests for multiple myeloma. Diagnostic study findings in multiple myeloma include...

Bence JOnes protein in the urine

The nurse cares for a client with acute myeloid leukemia with severe bone pain. What pathophysiological concept does the nurse understand is the reason for the client's pain? Abnormal blood cells deposit in small vessels. Bone marrow expands. Lymph nodes expand. Abnormal blood cells crystalize.

Bone marrow expands. Explanation: In acute myeloid leukemia, bone pain is caused when the bone marrow expands.

The Oncology Advanced Practice Nurse (APN) is assessing a client with leukemia. How would the Oncology APN assess for enlargement and tenderness over the liver and spleen?

By palpating the abdomen In a client with leukemia, the nurse palpates the abdomen to detect enlargement and tenderness over the liver and spleen. The nurse reviews laboratory test results to note the number and types of blood cells. The nurse may calculate the absolute neutrophil count to determine the client's potential for infection.

A 48-year-old female recently diagnosed with leukemia presents with increased immature lymphocytes, decreased granulocytes, and normal erythrocytes. The client most likely has which type of leukemia?

Chronic lymphocytic leukemia

A nurse reviews the laboratory results of a client with polycythemia vera. Which findings will the nurse find? Select all that apply.

Decreased erythropoietin Increased hemoglobin

A pt with mutiple myeloma is complaining about pain. What instructions will the nurse give the pt to help to reduce pain during activivty?

Do not lift more than 10 lbs

A client with multiple myeloma is complaining about pain. What instructions will the nurse give the client to help to reduce pain during activity?

Do not lift more than 10 pounds.

The nurse is interacting with a family that has been caring for a client with cancer for several months. What are the best interventions to assist in relieving caregiver stress in this family? Select all that apply.

Educate the family about medications and side effects. Allow family members to express feelings. Suggest support for household maintenance.

The nurse is interacting with a family that has been caring for a client with cancer for several months. What are the best interventions to assist in relieving caregiver stress in this family? Select all that apply.

Educate the family about medications and side effects. Allow family members to express feelings. Suggest support for household maintenance.

A client with multiple myeloma reports uncomfortable muscle cramping. Which nursing interventions will the nurse implement in response to the client's report of symptoms? Select all that apply.

Encourage hydration Encourage ambulation

A client is suspected of having leukemia and is having a series of laboratory and diagnostic studies performed. What does the nurse recognize as the hallmark signs of leukemia? Select all that apply.

Frequent infections Fatigue from anemia Easy bruising

When assessing a female client with a disorder of the hematopoietic or the lymphatic system, which assessment is most essential?

Health history, such as bleeding, fatigue, or fainting

A nurse is caring for a client with multiple myeloma. Which laboratory value is the nurse most likely to see?

Hypercalcemia

A patient with AML is having aggressive chemotherapy to attempt to achieve remission. The patient is aware that hospitalization will be necessary for several weeks. What type of therapy will the nurse explain that the patient will receive? - Standard therapy - Supportive therapy - Antimicrobial therapy - Induction therapy

Induction therapy Despite advances in understanding of the biology of AML, substantive advances in treatment response rates and survival rates have not occurred for decades, with the exception of advances made in treating APL (see later discussion). Even for patients with subtypes that have not benefited from advances in treatment, cure is still possible. The overall objective of treatment is to achieve complete remission, in which there is no evidence of residual leukemia in the bone marrow. Attempts are made to achieve remission by the aggressive administration of chemotherapy, called induction therapy, which usually requires hospitalization for several weeks.

A nurse cares for a client with myelodysplastic syndrome (MDS). Which assessment finding does the nurse recognize is the most common finding with this condition?

Macrocytic anemia Macrocytic anemia is the most common symptom of MDS.

A client has been diagnosed with polycythemia vera. What is the best instruction for the nurse to give to this client? Take a daily multivitamin with iron supplement Maintain adequate blood pressure control Drink alcohol to decrease blood viscosity Bath in tepid or cool water to control itching

Maintain adequate blood pressure control Explanation: The client with polycythemia vera needs to control blood pressure, because of the increased risk for thrombosis or hemorrhage. Iron supplements can stimulate red blood cell production. Ingestion of alcohol may cause bleeding. Bathing in cool or tepid water may control itching, but this is not as high a priority as preventing thrombosis or hemorrhage.

A pt has completed induction therapy and has diaarhea and severe mucositis. What is the appropriate nursing goal?

Maintain nutrition

A client has completed induction therapy and has diarrhea and severe mucositis. What is the appropriate nursing goal?

Maintain nutrition.

The nurse is caring for a client with multiple myeloma. Why would it be important to assess this client for fractures?

Osteoclasts break down bone cells so pathologic fractures occur.

The nurse is caring for a client with multiple myeloma. Why would it be important to assess this client for fractures? Osteopathic tumors destroy bone causing fractures. Osteoclasts break down bone cells so pathologic fractures occur. Osteolytic activating factor weakens bones producing fractures. Osteosarcomas form producing pathologic fractures.

Osteoclasts break down bone cells so pathologic fractures occur. Explanation: The abnormal plasma cells proliferate in the bone marrow, where they release osteoclast-activating factor. This in turn causes osteoclasts to break down bone cells, resulting in increased blood calcium and pathologic fractures. The plasma cells also form single or multiple osteolytic (bone-destroying) tumors that produce a 'punched-out' or 'honeycombed' appearance in bones such as the spine, ribs, skull, pelvis, femurs, clavicles, and scapulae. Weakened vertebrae lead to compression of the spine accompanied by significant pain. Options A, C, and D are distractors for this question.

Which term refers to an abnormal decrease in white blood cells, red blood cells, and platelets? - Anemia - Thrombocytopenia - Pancytopenia - Leukopenia

Pancytopenia Pancytopenia may be congenital or acquired. Anemia refers to decreased red cell mass. Leukopenia refers to a less-than-normal amount of WBCs in circulation. Thrombocytopenia refers to a lower-than-normal platelet count.

A patient with a diagnosis of immune thrombocytopenic purpura (ITP) is currently receiving IVIG for the treatment of her health condition. The nurse who is providing this patient's care is aware that ITP is a consequence of: - Platelet destruction and impaired platelet production resulting from an autoimmune process - Impaired liver function and the sequestering of platelets by hepatocytes - Hemolysis of platelets in individuals who lack immunity to the Epstein-Barr virus - Inappropriate platelet aggregation on the walls of the great vessels

Platelet destruction and impaired platelet production resulting from an autoimmune process Although the precise cause of ITP remains unknown, the platelet count is decreased by a combination of autoantibody-mediated platelet destruction and impaired platelet production secondary to autoantibody effects on the megakaryocyte. Viruses, impaired liver function, and inappropriate platelet aggregation are not dimensions of the etiology of ITP.

A nurse is caring for an asymptomatic client with acute myelogenous leukemia. The client has a total white blood cell (WBC) count of 0 ?l, a platelet count of 3,000 mm2, and a hemoglobin level of 9 mg/dl. He has a single lumen central venous catheter in place and the physician has ordered the nurse to administer imipenem cilastatin (Primaxin) 500 mg every 8 hours, transfuse 1 unit packed red blood cells (RBCs), give amphotericin B (Fungizone) 40 mg I.V. over 4 hours, and transfuse 2 pheresis units of platelets. In what order should the nurse infuse these medications and blood products?

Platelets, imipenem cilastatin, amphotericin B, packed RBCs

You are assisting your client with mulitple myeloma to ambulate. What is the most important nursing diagnosis to help prevent fractures in this client?

Safety

The nurse is assisting the client with multiple myeloma to ambulate. What is the most important nursing diagnosis to help prevent fractures in this client?

Safety Safety is paramount because any injury, no matter how slight, can result in a fracture.

A client with multiple myelo9ma presents to the ED complaining of excessive thirst and constipation. His family members report that he has been confused for the last day. Which lab value is most likely responsible for this clinet's symptoms?

Sercum calcium level of 133 mEq/L

A client with polycythemia vera reports gouty arthritis symptoms in the toes and fingers. What is the nurse's best understanding of the pathophysiological reason for this symptom?

The dead red blood cells release excess uric acid. There is a rapid proliferation of red blood cells from the marrow in polycythemia vera. However, these red blood cells die sooner than normal and the dead red blood cells release potassium and uric acid. This build up of uric acid in the blood leads to gouty arthritis symptoms.

The nurse is caring for a patient who will begin taking long-term biphosphate therapy. Why is it important for the nurse to encourage the patient to receive a thorough evaluation of dentition, including panoramic dental x-rays?

The patient can develop osteonecrosis of the jaw.

Indolent neoplasm

a slow-growing cancer that often remains localized or causes few symptoms

The nurse is teaching a patient about the development of leukemia. What statement should be included in the teaching plan? a) "Chronic leukemia develops slowly." b) "Acute leukemia develops slowly." c) "In acute leukemia there are not many undifferentiated cells." d) "In chronic leukemia, the majority of leukocytes are mature."

a) "Chronic leukemia develops slowly." Explanation: Chronic leukemia develops slowly, and the majority of leukocytes are still maturing. Acute leukemia develops quickly and the majority of leukocytes are undifferentiated cells.

Pancytopenia

abnormal decrease in WBCs, RBCs, and platelets

A patient is taking hydroxyurea for the treatment of primary myelofibrosis. While the patient is taking this medication, what will the nurse monitor to determine effectiveness? a) Blood urea nitrogen (BUN) and creatinine levels b) Leukocyte and platelet count c) Aspartate aminotransferase (AST) and alanine transaminase (ALT) levels d) Hemoglobin and hematocrit

b) Leukocyte and platelet count Hydroxyurea is often used in patients with primary myelofibrosis to control high leukocyte and platelet counts and to reduce the size of the spleen.

What type of cancer is the most common type of secondary malignancy in patients with Hodgkin's disease? a) Breast b) Lung c) Colon d) Bone

b) Lung Lung cancer is the most common type of secondary malignancy in patients with Hodgkin's disease, particularly following combination chemotherapy and radiation. Breast, colon, and bone are not the most common type of secondary malignancy.

A patient has completed induction therapy and has diarrhoea and severe mucositis. What is the appropriate nursing goal? a) Place client in reverse isolation. b) Maintain nutrition. c) Address issues of negative body image. d) Administer pain medication.

b) Maintain nutrition. Maintaining nutrition is the most important goal after induction therapy because the patient experiences severe diarrhea and can easily become nutritionally deficient as well as develop fluid and electrolyte imbalance. The patient is most likely not in pain at this point, and this is an intervention not a goal.

A patient who is undergoing chemotherapy for AML complains of pain in his lower back. What is the nurse's first action? a) Place heating pads on the patient's back. b) Refer the client to a chiropractor. c) Assess renal function. d) Administer pain medication, as ordered.

c) Assess renal function. Explanation: Chemotherapy results in the destruction of cells and tumor lysis syndrome. There is an increase in uric acid and phosphorus levels and the patient is susceptible to renal failure. The nurse should assess renal function if the patient complains of lower back pain as this could be indicative of a kidney stone formation. Heating pads, pain medication, and referrals could be instituted once the cause of the pain is determined. The priority is further assessment to rule out priority problems.

A nurse assesses a patient who has been diagnosed with DIC. Which of the following indicators are consistent with this diagnosis? Select all that apply. a) Capillary fill time <3 seconds b) Polyuria c) Increased blood urea nitrogen (BUN) and creatinine d) Cyanosis in the extremities e) Dyspnea and hypoxia f) Increased breath sounds

c) Increased blood urea nitrogen (BUN) and creatinine d) Cyanosis in the extremities e) Dyspnea and hypoxia Urine output would be decreased in DIC, and capillary fill time would be more than 3 seconds; breath sounds would be decreased. Refer to Table 20-4 in the text.

A client was admitted to the hospital with a pathologic pelvic fracture. The client informs the nurse that he has been having a strange pain in the pelvic area for a couple of weeks that was getting worse with activity prior to the fracture. What does the nurse suspect may be occurring based on these symptoms? a) Hemolytic anemia b) Leukemia c) Multiple myeloma d) Polycythemia vera

c) Multiple myeloma The first symptom usually is vague pain in the pelvis, spine, or ribs. As the disease progresses, the pain becomes more severe and localized. The pain intensifies with activity and is relieved by rest. When tumors replace bone marrow, pathologic fractures develop. Hemolytic anemia does not result in pathologic fractures nor does polycythemia vera or leukemia.

What assessment findings best indicate that the patient has recovered from induction therapy? a) Absence of bone pain b) No evidence of edema c) Neutrophil and platelet counts within normal limits d) Neutrophil and platelet counts within normal limits

c) Neutrophil and platelet counts within normal limits Explanation: Recovery from induction therapy is indicated when the neutrophil and platelet counts have returned to normal and any infection has resolved. Stable vital signs, lack of edema, and absence of pain are not indicative of recovery from induction therapy.

Hematopoiesis

complex process of the formation and maturation of blood cells

Thrombocytosis

higher-than-normal platelet count

Reticulocytes

slightly immature RBCs, usually only 1% of total circulating RBCs

Common symptoms seen in polycythemia vera include a ruddy complexion and ____________ from increased blood volume and viscosity.

splenomegaly

Petechiae

tiny capillary hemorrhages

A nurse assesses a patient who has been diagnosed with DIC. Which of the following indicators are consistent with this diagnosis? Select all that apply. - Capillary fill time <3 seconds - Increased breath sounds - Cyanosis in the extremities - Polyuria - Increased blood urea nitrogen (BUN) and creatinine - Dyspnea and hypoxia

- Increased blood urea nitrogen (BUN) and creatinine - Dyspnea and hypoxia - Cyanosis in the extremities Urine output would be decreased in DIC, and capillary fill time would be more than 3 seconds; breath sounds would be decreased. Refer to Table 20-4 in the text.

Which of the following are complications related to polycythemia vera (PV)? Select all that apply.

-CVA -MI -Ulcers -Hematuria Patients with PV are at increased risk for thromboses resulting in a CVA or myocardial infarction. Bleeding can be significant and can occur in the form of nosebleeds, ulcers, frank gastrointestinal bleeding, and intracranial hemorrhage. Splenomegaly is a clinical manifestation of PV, not a complication.

Place the pathophysiology of multiple myeloma in the correct order.

-Proliferation of abnormal plasma cells -Release of osteoclast-activating factor -Break down and removal of bone cells -Increased blood calcium levels The pathophysiology of multiple myeloma is as follows: Proliferation of abnormal plasma cells, release of osteoclast-activating factor, break down and removal of bone cells, increased blood calcium levels.

The nurse cares for an older adult client with unprovoked back pain and increased serum protein. Which hematologic neoplasm does the nurse suspect the client has? A.) Chronic myeloid leukemia B.) Multiple myeloma C.) Hodgkin lymphoma D.) Non-Hodgkin lymphoma

Answer: B.) Multiple myeloma

A client is recieving chemotherapy for acute myeloid leukemia and has poor nutritional intake. The first action of the nurse is to...

Ask, "are you experiencing nausea?"

A client presents with peripheral neuropathy and hypoesthesia of the feet. What is the best nursing intervention?

Assess for signs of injury.

The nurse cares for a client with acute myeloid leukemia with severe bone pain. What pathophysiological concept does the nurse understand is the reason for the client's pain?

Bone marrow expands.

A nurse cares for a client with early Hodgkin lymphoma. While assessing the client, the nurse will most likely find painless enlargement of which lymph node?

Cervical

A nurse cares for a client with early Hodgkin lymphoma. While assessing the client, the nurse will most likely find painless enlargement of which lymph node? Axillary Cervical Inguinal Popliteal

Cervical Explanation: Non painful swelling of the cervical lymph nodes is the earliest symptom of Hodgkin lymphoma.

A home care nurse is caring for a client with multiple myeloma. Which nursing interventions are appropriate for this client? Select all that apply.

Delay position changes and bathing if the client is experiencing pain. Instruct the client to avoid activities that may cause injury. Monitor renal function

A client with multiple myeloma is complaining about pain. What instructions will the nurse give the client to help to reduce pain during activity?

Do not lift more than 10 pounds. The client with multiple myeloma needs education about activity instructions, such as lifting no more than 10 pounds and using proper body mechanics. Braces may be needed. The clent should be active and would not be instructed to stay in bed or limit activity, as he or she would become very stiff. Limiting fluids would be contraindicated; the client needs to remain well hydrated.

A client who is being treated for AML has bruises on both legs. What is the nurse's most appropriate action?

Evaluate the client's platelet count.

A client who is being treated for AML has bruises on both legs. What is the nurse's mostappropriate action? Ask the client whether they have recently fallen. Evaluate the client's INR. Keep the client on bed rest. Evaluate the client's platelet count.

Evaluate the client's platelet count. Explanation: Complications of AML include bleeding. The risk of bleeding correlates with the level and duration of platelet deficiency. Major hemorrhages may develop when the platelet count drops to less than 10,000/mm3. The bleeding is usually unrelated to falling. Keeping the client on bed rest will not prevent bleeding when the client has a low platelet count. Assessment for other areas of bleeding is also a priority intervention.

A pt who is being treated for AML has bruises on both legs. What is the nurse's most appropriate action?

Evaluate the pt's platelet count

A patient is scheduled for a test to help confirm the diagnosis of acute myeloid leukemia (AML). Which of the following is the result that the nurse knows is consistent with the diagnosis?

Excess of immature blast cells

A nurse cares for a client with multiple myeloma who reports severe back pain that worsens throughout the day. What additional clinical symptoms will the nurse associate with the pathophysiology of the client's disease?

Excessive thirst

A client is suspected of having leukemia and is having a series of lab and diagnostic studies performed. What does the nurse recognize as the hallmark signs of leukemia?

Frequent infections, fatigue from anemia, and easy bruising

A nurse is caring for a client with multiple myeloma. Which laboratory value is the nurse most likely to see?

Hypercalcemia Calcium is released when bone is destroyed, causing hypercalcemia. Multiple myeloma doesn't affect potassium, sodium, or magnesium levels.

The nurse suspects a client's diagnosis of acute myeloid leukemia (AML) will be confirmed. What laboratory result is consistent with the medical diagnosis? Erythrocyte count of 5.8 m/L Platelet count of 300,000/mm3 Neutrophil reading of 60% Immature blast cells greater than 20%

Immature blast cells greater than 20% Explanation: Most clients with AML have too many white blood cells, not enough red blood cells, and not enough platelets. An excess of blast cells of greater than 20% is common with AML. The erythrocyte count of 5.8 m/L is normal. The platelet count of 300,000mm3 is normal. The neutrophil reading of 60% is normal.

A patient is taking hydroxyurea for the treatment of primary myelofibrosis. While the patient is taking this medication, what will the nurse monitor to determine effectiveness?

Leukocyte and platelet count

Which of the following terms refers to a form of WBC involved in immune response?

Lymphocyte

Which term refers to a form of white blood cell involved in immune response?

Lymphocyte

What assessment finding best indicates that the client has recovered from induction therapy?

Neutrophil and platelet counts within normal limits

What assessment findingbest indicates that the client has recovered from induction therapy?

Neutrophil and platelet counts within normal limits

What assessment findings best indicate that the pt has recovered from induction therapy?

Neutrophil and platelet counts within normal limits

What assessment findingbest indicates that the client has recovered from induction therapy?

Neutrophil and platelet counts within normal limits Recovery from induction therapy is indicated when the neutrophil and platelet counts have returned to normal and any infection has resolved. Stable vital signs, lack of edema, and absence of pain are not indicative of recovery from induction therapy.

Clients with multiple myeloma have abnormal plasma cells that proliferate in the bone marrow where they release osteoclast-activating factor, resulting in the formation of osteoclasts. Which of the following is the most common complication of the pathology resulting from this process?

Pathologic fractures Osteoclasts are cells that break down and remove bone cells, which results in increased blood calcium and pathologic fractures.

The hospitalized client is experiencing gastrointestinal bleeding with a platelets at 9,000/mm³. The client is receiving prednisone and azathioprine. What action will the nurse take?

Perform a neurologic assessment with vital signs.

The nurse is educating a pt taking imatinib mesylate (Gleevec) for treatment of leukemia. What should the nurse be sure to include when educating the pt on the best way to take the meds to optimize absorption?

Take antacids if needed for GI upset 2 hours after ftaking Gleevec

The nurse is assessing several clients. Which client does the nurse determine is most likely to have Hodgkin lymphoma?

The client with enlarged lymph nodes in the neck. Lymph node enlargement in Hodgkin lymphoma is not painful. The client with enlarged lymph nodes in the neck is most likely to have Hodgkin lymphoma if the enlarged nodes are painless. Sore throat is not a sign for this disorder.

A client with polycythemia vera reports gouty arthritis symptoms in the toes and fingers. What is the nurse's best understanding of the pathophysiological reason for this symptom? The dead red blood cells release excess uric acid. The dead red blood cells occlude the small vessels in the joints. Excess red blood cells produce extracellular toxins that build up. Excess red blood cells cause vascular injury in the joints.

The dead red blood cells release excess uric acid. Explanation: There is a rapid proliferation of red blood cells from the marrow in polycythemia vera. However, these red blood cells die sooner than normal and the dead red blood cells release potassium and uric acid. This build up of uric acid in the blood leads to gouty arthritis symptoms.

The nurse assesses a patient for late-stage chronic lymphocytic leukemia (CLL) by looking for what?

Thrombocytopenia.

T or F- A primary nursing diagnosis for a patient with acute leukemia is impaired nutritional intake due to the pain and discomfort associated with mucositis and stomatitis.

True

T or F- Chronic lymphocytic leukemia the most common form of leukemia and may require no treatment in the early stages.

True

A client has been diagnosed with multiple myeloma. Which of the following laboratory values should the nurse expect to find in a client with multiple myeloma? a) Decreased serum protein b) Increased urinary protein c) Polycythemia vera d) Decreased calcium level

b) Increased urinary protein A characteristic finding in multiple myeloma is protein in the urine. Other laboratory findings include increased serum protein, hypercalcemia, anemia, and hyperuricemia. Polycythemia vera is not found in multiple myeloma.

An elderly client is hospitalized for induction of chemotherapy to treat leukemia. The client reports fatigue to the nurse. What nursing intervention would best address the client's fatigue? a) Talk to the family about not visiting so the client can obtain rest. b) Provide sedentary activities only, such as watching television. c) Assist the client to sit in a chair for meals. d) Have the client maintain complete bedrest.

c) Assist the client to sit in a chair for meals. Fatigue is a common symptom with clients who have leukemia. Despite the fatigue, clients still need to maintain some physical activity. An example of physical activity is having the client sit in a chair for meals. The nurse does not want to encourage complete bedrest or sedentary activities, such as watching television, due to possible deconditioning. The nurse has not discussed with the client about limiting family visits. The client may want some family to visit.

A patient presents with peripheral neuropathy and hypothesia of the feet. What is the best nursing intervention? a) Keep the feet cool. b) Encourage ambulation. c) Have the client elevate his legs. d) Assess for signs of injury.

d) Assess for signs of injury. Explanation: A patient with hypothesia of the feet will have decreased sensation and numbness. The nurse should assess for signs of injury. If the patient has injured himself, he will not be able to feel it and this could lead to the development of infection. Ambulation will not help the patient and elevation of the legs may make the problem worse as blood flow to the feet would be decreased. Keeping the feet cold will also decrease blood flow.

White blood cell (WBC): synonym

leukocyte

Acute myeloid leukemia (AML) results from a defect in the hematopoietic stem cell that differentiates into what myeloid cells?

monocytes, granulocytes, erythrocytes, and platelets

Leukocyte

one of several cellular components of blood involved in defense of the body; subtypes include neutrophils, eosinophils, basophils, monocytes, and lymphocytes (synonym: white blood cell [WBC])

Apoptosis

programmed cell death

Cytokines

proteins produced by leukocytes that are vital to regulation of hematopoiesis, apoptosis, and immune responses

Following bone marrow aspiration of a 19-year-old client, analysis reveals more than 20% immature blast cells. Platelet counts are 9000/mm³. What nursing interventions should the nurse employ for the care of this client? Select all answers that apply. - Administer prescribed docusate (Colace) daily. - Recommend taking ibuprofen for mild aches and pains. - Apply pressure to venipuncture sites for 1 to 2 minutes. - Assess for mental state changes. - Discuss the withholding of oral contraceptives.

- Administer prescribed docusate (Colace) daily. - Assess for mental state changes. The client has leukemia with immature blast cells and an extremely low platelet count. The client is at increased risk for bleeding. Interventions that would address bleeding include assessing for mental status changes (because bleeding could occur in the brain) and administering stool softeners to prevent constipation (which would increase the risk of bleeding from the rectum). Oral contraceptives would be administered to induce amenorrhea. Ibuprofen would be avoided because this medication inhibits platelet function. The nurse is to apply pressure to venipuncture sites for 5 minutes.

A patient with polycythemia vera is complaining of severe itching. What triggers does the nurse know can cause this distressing symptom? (Select all that apply.) - Aspirin - Allergic reaction to the red blood cell increase - Alcohol consumption - Exposure to water of any temperature - Temperature change

- Alcohol consumption - Exposure to water of any temperature - Temperature change Pruritus is very common, occurring in up to 70% of patients with polycythemia vera (Saini, Patnaik & Tefferi, 2010) and is one of the most distressing symptoms of this disease. It is triggered by contact with temperature change, alcohol consumption, or, more typically, exposure to water of any temperature but seems to be worse with exposure to hot water.

The nurse is interacting with a family that has been caring for a client with cancer for several months. What are the best interventions to assist in relieving caregiver stress in this family? Select all that apply. - Suggest the family go to church more often. - Suggest support for household maintenance. - Suggest the prescription of antianxiety medications. - Educate the family about medications and side effects. - Allow family members to express feelings.

- Suggest support for household maintenance. - Educate the family about medications and side effects. - Allow family members to express feelings.

The nurse is interacting with a family that has been caring for a client with cancer for several months. What are the best interventions to assist in relieving caregiver stress in this family? Select all that apply.

-Educate the family about medications and side effects. -Allow family members to express feelings. -Suggest support for household maintenance. Family members benefit from increased education on what to expect. Allowing family members to express their feelings has also been shown to relieve stress. Supporting the caregiver and family with help in household duties will also help the overburdened family. Antianxiety medications and church attendance have not been shown to reduce caregiver stress.

After chemotherapy for AML, what interventions will best help to prevent renal complications? Select all that apply.

-Increase hydration. -Administer allopurinol. -Administer rasburicase. Increased uric acid and phosphorus concentrations after chemotherapy for AML can lead to renal calculi formation. Increasing hydration and administering allopurinol (a uricosuric) will help to eliminate the uric acid. Rasburicase is an enzyme that can also decrease uric acid. Administration of potassium is not indicated, as concentrations are elevated after chemotherapy. Exercise is not initially encouraged because the client could have weakness and cramping during this time.

The nurse is caring for a patient with Hodgkin lymphoma in the hospital and preparing discharge planning education. Knowing that this patient is at risk for the development of a second malignancy, what education would be beneficial to reduce the risk factors? (Select all that apply.)

-Reduce exposure to excessive sunlight -Smoking cessation -Decrease alcohol intake The potential development of a second malignancy should be addressed with the patient when initial treatment decisions are made. However, patients should also be told that Hodgkin lymphoma is often curable. The nurse should encourage patients to reduce other factors that increase the risk of developing second cancers, such as use of tobacco and alcohol and exposure to environmental carcinogens and excessive sunlight.

A client with polycythemia vera presents to the primary care clinic for an annual physical. Which health care provider prescriptions does the nurse recognize as preventive measures to prevent thrombosis in the client? Select all that apply.

-Statin therapy -Beta blocker therapy Aggressive management of atherosclerosis, by treating hypertension and hyperlipidemia, is important in diminishing the risk of thrombosis in the client with polycythemia vera. Statin therapy decreases cholesterol levels while beta blockers decrease blood pressure. Iron therapy would make the client worse, not better. Nitrates and anti-dysrhythmia drugs are not primarily used in the treatment of polycythemia vera.

Which of the following is the only curative treatment for chronic myeloid leukemia (CML)? - Idarubicin - Cytarabine - Allogeneic stem cell transplant - Imatinib

Allogeneic stem cell transplant Allogeneic stem cell transplantation remains the only curative treatment for CML. The efficacy of Imatinib as first-line treatment and the treatment-related mortality of stem cell transplant limits use of transplant to patients with high risk or relapsed disease, or in those patients who did not respond to therapy with TKI. Cytarabine and idarubicin are part of induction therapy for acute myeloid leukemia (AML).

A patient with AML is having aggressive chemotherapy to attempt to achieve remission. The patient is aware that hospitalization will be necessary for several weeks. What type of therapy will the nurse explain that the patient will receive? A.) Induction therapy B.) Supportive therapy C.) Antimicrobial therapy D.) Standard therapy

Answer: A.) Induction therapy Rationale: Despite advances in understanding of the biology of AML, substantive advances in treatment response rates and survival rates have not occurred for decades, with the exception of advances made in treating APL (see later discussion). Even for patients with subtypes that have not benefited from advances in treatment, cure is still possible. The overall objective of treatment is to achieve complete remission, in which there is no evidence of residual leukemia in the bone marrow. Attempts are made to achieve remission by the aggressive administration of chemotherapy, called induction therapy, which usually requires hospitalization for several weeks.

A nurse cares for a client with myelodysplastic syndrome (MDS). Which assessment finding does the nurse recognize is the most common finding with this condition? A.) Macrocytic anemia B.) Microcytic anemia C.) Proliferative anemia D.) Hemolytic anemia

Answer: A.) Macrocytic anemia

Which term refers to an abnormal decrease in white blood cells, red blood cells, and platelets? A.) Pancytopenia B.) Anemia C.) Leukopenia D.) Thrombocytopenia

Answer: A.) Pancytopenia

Clients with multiple myeloma have abnormal plasma cells that proliferate in the bone marrow where they release osteoclast-activating factor, resulting in the formation of osteoclasts. What is the most common complication of the pathology resulting from this process? A.) Pathologic fractures B.) Osteoporosis C.) Calcified bones D.) Increased mobility

Answer: A.) Pathologic fractures

A client with polycythemia vera reports gouty arthritis symptoms in the toes and fingers. What is the nurse's best understanding of the pathophysiological reason for this symptom? A.) The dead red blood cells release excess uric acid. B.) The dead red blood cells occlude the small vessels in the joints. C.) Excess red blood cells produce extracellular toxins that build up. D.) Excess red blood cells cause vascular injury in the joints.

Answer: A.) The dead red blood cells release excess uric acid.

A client with AML has pale mucous membranes and bruises on the legs. What is the primary nursing intervention? A.) Assess the client's skin. B.) Assess the client's hemoglobin and platelets. C.) Assess the client's pulse and blood pressure. D.) Check the client's history.

Answer: B.) Assess the client's hemoglobin and platelets.

The clinic nurse is caring for a client diagnosed with leukopenia. What does the nurse know this client has? A.) Too many erythrocytes B.) A decrease in granulocytes C.) A general reduction in all white blood cells D..) A general reduction in neutrophils and basophils

Answer: C.) A general reduction in all white blood cells

Which nursing intervention should be incorporated into the plan of care to manage the delayed clotting process in a client with leukemia? A.) Implement neutropenic precautions. B.) Eliminate direct contact with others who are infectious. C.) Apply prolonged pressure to needle sites or other sources of external bleeding. D.) Monitor temperature at least once per shift.

Answer: C.) Apply prolonged pressure to needle sites or other sources of external bleeding.

The nurse is performing an assessment on a patient with acute myeloid leukemia (AML) and observes multiple areas of ecchymosis and petechiae. What laboratory study should the nurse be concerned about? A.) WBC count of 4,200 cells/uL B.) Hematocrit of 38% C.) Platelet count of 9,000/mm3 D.) Creatinine level of 1.0 mg/dL

Answer: C.) Platelet count of 9,000/mm3 Rationale: Complications of AML include bleeding and infection, which are the major causes of death. The risk of bleeding correlates with the level and duration of platelet deficiency (thrombocytopenia). The low platelet count can cause ecchymoses (bruises) and petechiae. Major hemorrhages also may develop when the platelet count drops to less than 10,000/mm3.

The nurse is assessing a patent with polycythemia vera. What skin assessment data would the nurse determine is a normal finding for this patient? A.) Pale skin and mucous membranes B.) Bronze skin tone C.) Ruddy complexion D.) Jaundice skin and sclera

Answer: C.) Ruddy complexion Rationale: Polycythemia vera (sometimes called P vera), or primary polycythemia, is a proliferative disorder of the myeloid stem cells. Patients typically have a ruddy complexion and splenomegaly.

A client who is undergoing chemotherapy for AML reports pain in the low back. What is the nurse's first action? A.) Refer the client to a chiropractor. B.) Place heating pads on the client's back. C.) Administer pain medication, as ordered. D.) Assess renal function.

Answer: D.) Assess renal function. Rationale: Chemotherapy results in the destruction of cells and tumor lysis syndrome. Uric acid and phosphorus concentrations increase, and the client is susceptible to renal failure. The nurse should assess renal function if the client complains of low-back pain, as this could be indicative of kidney stone formation. Heating pads, pain medication, and referrals could be instituted once the cause of the pain is determined. The priority is further assessment to rule out important problems.

The nurse recognizes the clinical assessment of a patient with acute myeloid leukemia (AML) includes observing for signs of infection early. What nursing action will most likely help prevent infection? A.) Monitor the client's temperature every shift. B.) Maintain contact precautions. C.) Encourage increased fluid consumption. D.) Practice vigilant handwashing.

Answer: D.) Practice vigilant handwashing.

A nurse is caring for a client with multiple myeloma. Which nursing intervention is most appropriate for this client? A.) Monitoring respiratory status B.) Balancing rest and activity C.) Restricting fluid intake D.) Preventing bone injury

Answer: D.) Preventing bone injury

Which patient assessed by the nurse is most likely to develop myelodysplastic syndrome (MDS)? A.) A 24-year-old female taking oral contraceptives B.) A 40-year-old patient with a history of hypertension C.) A 52-year-old patient with acute kidney injury D.) A 72-year-old patient with a history of cancer

Answer:" D.) A 72-year-old patient with a history of cancer

The nurse is interacting with a family that has been caring for a client with cancer for several months. What are the best interventions to assist in relieving caregiver stress in this family? Select all that apply. - Educate the family about medications and side effects. - Allow family members to express feelings. - Suggest support for household maintenance. - Suggest the prescription of antianxiety medications. - Suggest the family go to church more often.

Answer; - Educate the family about medications and side effects. - Allow family members to express feelings. - Suggest support for household maintenance.

What assessment finding best indicates that the client has recovered from induction therapy? A.) Neutrophil and platelet counts within normal limits B.) Vital signs within normal ranges C.) No evidence of edema D.) Absence of bone pain

Answer; A.) Neutrophil and platelet counts within normal limits

Which of the following nursing interventions should be incorporated into the plan of care to manage the delayed clotting process in a pt with leukemia?

Apply prolonged pressure to needle sites or other sources of external bleeding

Which nursing intervention should be incorporated into the plan of care to manage the delayed clotting process in a client with leukemia?

Apply prolonged pressure to needle sites or other sources of external bleeding.

Which nursing intervention should be incorporated into the plan of care to manage the delayed clotting process in a client with leukemia? Implement neutropenic precautions. Eliminate direct contact with others who are infectious. Apply prolonged pressure to needle sites or other sources of external bleeding. Monitor temperature at least once per shift.

Apply prolonged pressure to needle sites or other sources of external bleeding. Explanation: For a client with leukemia, the nurse should apply prolonged pressure to needle sites or other sources of external bleeding. Reduced platelet production results in a delayed clotting process and increases the potential for hemorrhage. Implementing neutropenic precautions and eliminating direct contact with others are interventions to address the risk for infection.

Which nursing intervention should be incorporated into the plan of care to manage the delayed clotting process in a client with leukemia? - Apply prolonged pressure to needle sites or other sources of external bleeding. - Monitor temperature at least once per shift. - Eliminate direct contact with others who are infectious. - Implement neutropenic precautions.

Apply prolonged pressure to needle sites or other sources of external bleeding. For a client with leukemia, the nurse should apply prolonged pressure to needle sites or other sources of external bleeding. Reduced platelet production results in a delayed clotting process and increases the potential for hemorrhage. Implementing neutropenic precautions and eliminating direct contact with others are interventions to address the risk for infection.

A 50-year-old woman was recently diagnosed with non-Hodgkin's lymphoma (NHL) and has begun a treatment regimen that includes simultaneous radiation therapy and chemotherapy. The combination of severe symptoms and aggressive therapy has necessitated admission to the hospital. When providing care for this patient, which of the following actions should the nurse implement? - Applying standard precautions conscientiously to reduce the patient's risk of infection - Monitoring the patient's bowel pattern and facilitating a high-fiber diet - Encouraging frequent mobilization and independence in activities of daily living - Providing meticulous skin care and turning the patient at least once every 2 hours

Applying standard precautions conscientiously to reduce the patient's risk of infection Treatment for NHL creates a significant risk of infection, a threat that must be minimized when planning and implementing nursing care. This is a priority over ADLs in the short term. The patient does not have a significantly increased risk of skin breakdown or constipation, although the nurse would assess for each problem.

A client is receiving chemotherapy for acute myeloid leukemia and has poor nutritional intake. What is the first action the nurse should take? Ask, "Are you experiencing nausea?" Provide mouth care before each meal. Caution the client to chew carefully after administration of the prescribed lidocaine. Provide nutritional supplements in addition to a diet that has a soft texture and moderate temperature.

Ask, "Are you experiencing nausea?" Explanation: All these options are things the nurse can do to assist the client to obtain better nutrition. The nurse first needs to assess the reason for poor nutritional intake. It could be because of nausea, in which case the nurse would implement interventions to address the client's nausea.

Following bone marrow aspiration of a 19-year-old client, analysis reveals more than 20% immature blast cells. Platelet counts are 9000/mm³. What nursing interventions should the nurse employ for the care of this client?

Assess for mental state changes. Administer prescribed docusate (Colace) daily. The client has leukemia with immature blast cells and an extremely low platelet count. The client is at increased risk for bleeding. Interventions that would address bleeding include assessing for mental status changes (because bleeding could occur in the brain) and administering stool softeners to prevent constipation (which would increase the risk of bleeding from the rectum). Oral contraceptives would be administered to induce amenorrhea. Ibuprofen would be avoided because this medication inhibits platelet function. The nurse is to apply pressure to venipuncture sites for 5 minutes.

A client presents with peripheral neuropathy and hypothesia of the feet. What is the best nursing intervention?

Assess for signs of injury.

A client presents with peripheral neuropathy and hypothesia of the feet. What is the best nursing intervention? - Assess for signs of injury. - Keep the feet cool. - Elevate the client's legs. - Encourage ambulation.

Assess for signs of injury. A client with hypothesia of the feet will have decreased sensation and numbness. The nurse should assess for signs of injury. If the client is injured, he or she will not be able to feel it; this could lead to the development of infection. Ambulation will not help the client, and elevating the legs may make the problem worse, as blood flow to the feet would be decreased. Keeping the feet cold will also decrease blood flow.

A client who is undergoing chemotherapy for AML reports pain in the low back. What is the nurse's first action? Refer the client to a chiropractor. Place heating pads on the client's back. Administer pain medication, as ordered. Assess renal function.

Assess renal function. Explanation: Chemotherapy results in the destruction of cells and tumor lysis syndrome. Uric acid and phosphorus concentrations increase, and the client is susceptible to renal failure. The nurse should assess renal function if the client complains of low-back pain, as this could be indicative of kidney stone formation. Heating pads, pain medication, and referrals could be instituted once the cause of the pain is determined. The priority is further assessment to rule out important problems.

What interventions are most appropriate for the nurse to include in the plan of care for a client at risk for infection? Select all that apply. Assess skin and mucus membranes every shift. Auscultate lung sounds every shift and as needed. Place fresh flowers on a shelf on the opposite wall from the client. Encourage the client to take deep breaths every 4 hours while awake. Provide oral hygiene once daily.

Assess skin and mucus membranes every shift. Auscultate lung sounds every shift and as needed. Encourage the client to take deep breaths every 4 hours while awake. Explanation: Interventions for risk for infection include assessing skin and mucus membranes every shift, auscultating lung sounds every shift and as needed, and encouraging deep breaths every 4 hours while the client is awake. No fresh flowers are allowed in the room because of germs found in stagnant water. Oral hygiene should be provided after meals and every 4 hours while the client is awake.

An elderly client is hospitalized for induction of chemotherapy to treat leukemia. The client reports fatigue to the nurse. What nursing intervention would best address the client's fatigue? Have the client maintain complete bedrest. Assist the client to sit in a chair for meals. Talk to the family about not visiting so the client can obtain rest. Provide sedentary activities only, such as watching television.

Assist the client to sit in a chair for meals. Explanation: Fatigue is a common symptom with clients who have leukemia. Despite the fatigue, clients still need to maintain some physical activity. An example of physical activity is having the client sit in a chair for meals. The nurse does not want to encourage complete bedrest or sedentary activities, such as watching television, due to possible deconditioning. The nurse has not discussed with the client about limiting family visits. The client may want some family to visit.

A client is undergoing tests for multiple myeloma. Diagnostic study findings in multiple myeloma include:

Bence Jones protein in the urine Presence of Bence Jones protein in the urine almost always confirms multiple myeloma; however, absence of the protein doesn't rule out the disease. Serum creatinine level may be increased (above 1.2 mg/dl in men and 0.9 mg/dl in women). Serum calcium levels are above 10.2 mg/dl in multiple myeloma because calcium is lost from the bone and reabsorbed in the serum. The serum protein level is increased in multiple myeloma, not decreased.

A client is undergoing tests for multiple myeloma. Diagnostic study findings in multiple myeloma include: - Serum calcium level of 7.5 mg/dl - Serum creatinine level 0.5 mg/dl - Bence Jones protein in the urine - Serum protein level 5.8 g/dl

Bence Jones protein in the urine Presence of Bence Jones protein in the urine almost always confirms multiple myeloma; however, absence of the protein doesn't rule out the disease. Serum creatinine level may be increased (above 1.2 mg/dl in men and 0.9 mg/dl in women). Serum calcium levels are above 10.2 mg/dl in multiple myeloma because calcium is lost from the bone and reabsorbed in the serum. The serum protein level is increased in multiple myeloma, not decreased.

A nurse practitioner suspects that a pt has multiple myeloma based on his major presenting symptom and the analysis of his lab results. What is the classic symptom of the disease?

Bone pain the back of the ribs

The nurse is assessing a client with leukemia. How would the nurse assess for enlargement and tenderness over the liver and spleen?

By palpating the abdomen

What are complications of polycythemia vera (PV)?

CVA, MI, Ulcers, Hematuria

A client with acute myeloid leukemia (AML) receiving chemotherapy is treated for an acute renal injury. What is the nurse's best understanding of the pathophysiological reason behind the client's injury?

Chemotherapy causes an increase in kidney stone formation. Massive leukemic cell destruction from chemotherapy results in the release of intracellular electrolytes and fluids into the systemic circulation. This causes an increase in uric acid levels, potassium, and phosphate (also known as tumor lysis). The increase in uric acid predisposes the client to the development of kidney stones and increases the risk for renal injury.

The nursing instructor is talking with their clinical group about coagulopathies. How should the instructor define coagulopathies? - Coagulopathies are bleeding disorders that are characterized by a deficiency of globulins in the plasma. - Coagulopathies are bleeding disorders that involve platelets or clotting factors. - Coagulopathies are bleeding disorders that are characterized by abnormalities in the numbers and types of red blood cells in the body. - Coagulopathies are bleeding disorders that involve the destruction of stem cells in the bone marrow.

Coagulopathies are bleeding disorders that involve platelets or clotting factors. Coagulopathies are bleeding disorders that involve platelets or clotting factors. Coagulopathies do not involve the numbers and types of red blood cells. They are not characterized by a deficiency of globulins in the plasma and they do not involve the destruction of stem cells in the bone marrow.

A home care nurse is caring for a client with multiple myeloma. Which nursing interventions are appropriate for this client?

Delay position changes and bathing if the client is experiencing pain. Instruct the client to avoid activities that may cause injury. Monitor renal function Pain can become quite severe. Delay position changes and bathing until analgesic has reached peak concentration level and the client is experiencing maximum pain relief. Safety is paramount because any injury, no matter how slight, can result in a fracture. The nurse assists the client with ambulation because immobility can worsen loss of calcium from the bone. The nurse provides up to 4000 mL of fluid to prevent renal damage from hypercalcemia and precipitation of protein in the renal tubules.

A client is receiving radiation therapy for lesions in the abdomen from non-Hodgkin's lymphoma. Because of the effects of the radiation treatments, what will the nurse assess for? Adventitous lung sounds Hair loss Diarrheal stools Laryngeal edema

Diarrheal stools Explanation: Side effects of radiation therapy are limited to the area being irradiated. Clients who have abdominal radiation therapy may experience diarrhoea. If the lesions were in the upper chest, then the client may experience adventitious lung sounds or laryngeal oedema as side effects. Hair loss is associated more with chemotherapy than radiation therapy.

The nurse is interacting with a family that has been caring for a client with cancer for several months. What are the best interventions to assist in relieving caregiver stress in this family? Select all that apply. Educate the family about medications and side effects. Allow family members to express feelings. Suggest support for household maintenance. Suggest the prescription of antianxiety medications. Suggest the family go to church more often.

Educate the family about medications and side effects. Allow family members to express feelings. Suggest support for household maintenance. Explanation: Family members benefit from increased education on what to expect. Allowing family members to express their feelings has also been shown to relieve stress. Supporting the caregiver and family with help in household duties will also help the overburdened family. Antianxiety medications and church attendance have not been shown to reduce caregiver stress.

Which precautions should a nurse include in the care plan for a client with leukemia and neutropenia? Have the client use a soft toothbrush and electric razor, avoid using enemas, and watch for signs of bleeding. Put on a mask, gown, and gloves when entering the client's room. Provide a clear liquid, low-sodium diet. Eliminate fresh fruits and vegetables, avoid using enemas, and practice frequent hand washing.

Eliminate fresh fruits and vegetables, avoid using enemas, and practice frequent hand washing. Explanation: Neutropenia occurs when the absolute neutrophil count falls below 1,000/mm3, reflecting a severe risk of infection. The nurse should provide a low-bacterial diet, which means eliminating fresh fruits and vegetables, avoiding invasive procedures such as enemas, and practicing frequent hand washing. Using a soft toothbrush, avoiding straight-edged razors and enemas, and monitoring for bleeding are precautions for clients with thrombocytopenia. Putting on a mask, gown, and gloves when entering the client's room are reverse isolation measures. A neutropenic client doesn't need a clear liquid diet or sodium restrictions.

Which precautions should a nurse include in the care plan for a client with leukemia and neutropenia? - Provide a clear liquid, low-sodium diet. - Put on a mask, gown, and gloves when entering the client's room. - Eliminate fresh fruits and vegetables, avoid using enemas, and practice frequent hand washing. - Have the client use a soft toothbrush and electric razor, avoid using enemas, and watch for signs of bleeding.

Eliminate fresh fruits and vegetables, avoid using enemas, and practice frequent hand washing. Neutropenia occurs when the absolute neutrophil count falls below 1,000/mm3, reflecting a severe risk of infection. The nurse should provide a low-bacterial diet, which means eliminating fresh fruits and vegetables, avoiding invasive procedures such as enemas, and practicing frequent hand washing. Using a soft toothbrush, avoiding straight-edged razors and enemas, and monitoring for bleeding are precautions for clients with thrombocytopenia. Putting on a mask, gown, and gloves when entering the client's room are reverse isolation measures. A neutropenic client doesn't need a clear liquid diet or sodium restrictions.

A client with leukemia has developed a cough and increased fatigue. What is the primary nursing intervention? Evaluate the client for potential infection. Administer an antitussive. Place a cooling blanket on the client. Medicate the client to relieve pain.

Evaluate the client for potential infection. Explanation: The client with leukemia has a lack of mature and normal granulocytes to fight infection. For this reason, the client is susceptible to infection. The primary nursing intervention is to evaluate for potential infection if the client has a cough and increased fatigue. Administering an antitussive would not be appropriate before determining the cause of the cough. A cooling blanket would not be needed if the client does not have a fever. Medicating the client to relieve pain would come after the assessment phase.

A client with leukemia has developed a cough and increased fatigue. What is the primary nursing intervention? - Medicate the client to relieve pain. - Place a cooling blanket on the client. - Evaluate the client for potential infection. - Administer an antitussive.

Evaluate the client for potential infection. The client with leukemia has a lack of mature and normal granulocytes to fight infection. For this reason, the client is susceptible to infection. The primary nursing intervention is to evaluate for potential infection if the client has a cough and increased fatigue. Administering an antitussive would not be appropriate before determining the cause of the cough. A cooling blanket would not be needed if the client does not have a fever. Medicating the client to relieve pain would come after the assessment phase.

A client who is being treated for AML has bruises on both legs. What is the nurse's most appropriate action? - Ask the client whether they have recently fallen. - Keep the client on bed rest. - Evaluate the client's INR. - Evaluate the client's platelet count.

Evaluate the client's platelet count. Complications of AML include bleeding. The risk of bleeding correlates with the level and duration of platelet deficiency. Major hemorrhages may develop when the platelet count drops to less than 10,000/mm3. The bleeding is usually unrelated to falling. Keeping the client on bed rest will not prevent bleeding when the client has a low platelet count. Assessment for other areas of bleeding is also a priority intervention.

A pt is scheduled for a test to help confirm the diagnosis of acute myeloid leukemia (AML). What result does the nurse knows is consistent with the diagnosis?

Excess of immature blast cells

A nurse prepares a client for a bone marrow biopsy who is suspected of having acute myeloid leukemia. What results from the bone marrow biopsy does the nurse expect?

Excess of immature leukocytes

A nurse prepares a client for a bone marrow biopsy who is suspected of having acute myeloid leukemia. What results from the bone marrow biopsy does the nurse expect? Excess of immature leukocytes Excess of immature erythrocytes Deficiency of neutrophils Deficiency of erythrocytes

Excess of immature leukocytes Explanation: The bone marrow biopsy of a client with acute myeloid leukemia will reveal an excess of immature leukocytes.

A nurse cares for a client with multiple myeloma who reports severe back pain that worsens throughout the day. What additional clinical symptoms will the nurse associate with the pathophysiology of the client's disease? Diarrhea Excessive thirst Polyuria Fluid volume excess

Excessive thirst Explanation: Bone pain in multiple myeloma results from bone breakdown. As a result of the breakdown, ionized calcium is released into the blood causing hypercalcemia. Symptoms of hypercalcemia include excessive thirst, dehydration, and constipation.

A client is suspected of having leukemia and is having a series of laboratory and diagnostic studies performed. What does the nurse recognize as the hallmark signs of leukemia? Select all that apply. Diarrhea Nausea and vomiting Frequent infections Fatigue from anemia Easy bruising

Frequent infections Fatigue from anemia Easy bruising Explanation: Infections, fatigue from anemia, and easy bruising are hallmarks of leukemia. At the onset of leukemia, particularly in acute lymphocytic leukemia (ALL), a fever is present, the spleen and lymph nodes enlarge, and internal or external bleeding develops. Diarrhea and nausea and vomiting are not the hallmark signs of leukemia and can be indicators in many illnesses and gastrointestinal disorders.

A patient with acute myeloid leukemia (AML) is having hematopoietic stem cell transplantation (HSCT) with radiation therapy. In which complication do the donor's lymphocytes recognize the patient's body as foreign and set up reactions to attack the foreign host?

Graft-versus-host disease

A pt with acute myeloid leukemia (AML) is having hematopoietic stem cell transplantaion (HSCT) with radiation therapy. In which complication do the donor's lymphocytes recognize the pt's body as foreign and set up reactions to attack the foreign host?

Graft-versus-host disease

A patient with acute myeloid leukemia (AML) is having hematopoietic stem cell transplantation (HSCT) with radiation therapy. In which complication do the donor's lymphocytes recognize the patient's body as foreign and set up reactions to attack the foreign host? Acute respiratory distress syndrome Graft-versus-host disease Remission Bone marrow depression

Graft-versus-host disease Explanation: Patients who undergo HSCT have a significant risk of infection, graft-versus host disease (in which the donor's lymphocytes [graft] recognize the patient's body as "foreign" and set up reactions to attack the foreign host), and other complications.

A patient with acute myeloid leukemia (AML) is having hematopoietic stem cell transplantation (HSCT) with radiation therapy. In which complication do the donor's lymphocytes recognize the patient's body as foreign and set up reactions to attack the foreign host? - Acute respiratory distress syndrome - Graft-versus-host disease - Remission - Bone marrow depression

Graft-versus-host disease Patients who undergo HSCT have a significant risk of infection, graft-versus host disease (in which the donor's lymphocytes [graft] recognize the patient's body as "foreign" and set up reactions to attack the foreign host), and other complications.

A nurse is caring for a client with multiple myeloma. Which laboratory value is the nurse most likely to see? - Hypermagnesemia - Hypernatremia - Hyperkalemia - Hypercalcemia

Hypercalcemia Calcium is released when bone is destroyed, causing hypercalcemia. Multiple myeloma doesn't affect potassium, sodium, or magnesium levels.

A nurse is caring for a client with multiple myeloma. Which laboratory value is the nurse most likely to see? Hypercalcemia Hyperkalemia Hypernatremia Hypermagnesemia

Hypercalcemia Explanation: Calcium is released when bone is destroyed, causing hypercalcemia. Multiple myeloma doesn't affect potassium, sodium, or magnesium levels.

A nurse plans care for a client with multiple myeloma. Using the CRAB acronym for symptoms associated with this disease, which clinical features does the nurse expect to find upon assessment of the client? Select all that apply.

Hypercalcemia Renal insufficiency Anemia Bone lesions

A nurse plans care for a client with multiple myeloma. Using the CRAB acronym for symptoms associated with this disease, which clinical features does the nurse expect to find upon assessment of the client? Select all that apply. Hypercalcemia Renal insufficiency Anemia Bone lesions Acidosis

Hypercalcemia Renal insufficiency Anemia Bone lesions Explanation: The acronym CRAB is used to describe the combined pathologic effects of multiple myeloma and include: calcium levels elevated (hypercalcemia), renal insufficiency, anemia, bone lesions. Acidosis is not part of the acronym used to describe the pathologic effects of the disease.

The nurse is currently planning the care of a patient with multiple myeloma who is experiencing bone destruction. When reviewing the patient's most recent blood work, what value would the nurse pay particular attention to? - Hypercalcemia - Elevated red blood cell (RBC) count - Hyperproteinemia - Elevated serum viscosity

Hypercalcemia Hypercalcemia may occur when bone destruction occurs due to the disease process. Elevated serum viscosity occurs because plasma cells excrete excess immunoglobulin. RBC count will be decreased. Hyperproteinemia would not be present.

A patient with AML is having aggressive chemotherapy to attempt to achieve remission. The patient is aware that hospitalization will be necessary for several weeks. What type of therapy will the nurse explain that the patient will receive? Induction therapy Supportive therapy Antimicrobial therapy Standard therapy

Induction therapy Explanation: Despite advances in understanding of the biology of AML, substantive advances in treatment response rates and survival rates have not occurred for decades, with the exception of advances made in treating APL (see later discussion). Even for patients with subtypes that have not benefited from advances in treatment, cure is still possible. The overall objective of treatment is to achieve complete remission, in which there is no evidence of residual leukemia in the bone marrow. Attempts are made to achieve remission by the aggressive administration of chemotherapy, called induction therapy, which usually requires hospitalization for several weeks.

The nurse is administering packed red blood cell (RBC) transfusions for a patient with myelodysplastic syndrome (MDS). The patient has had several transfusions and is likely to receive several more. What is a priority for the nurse to monitor related to the transfusions? Creatinine and blood urea nitrogen (BUN) levels Iron levels Magnesium levels Potassium levels

Iron levels Explanation: For most patients with MDS, transfusions of RBCs may be required to control the anemia and its symptoms. These patients can develop iron overload from the repeated transfusions; this risk can be diminished with prompt initiation of chelation therapy (see following Nursing Management section).

A pt is taking hydroxyurea for the treatment of primary myelofibrosis. While the pt is taking this med, what will the nurse monitor to determine effectiveness?

Leukocyte and platelet count

A patient is taking hydroxyurea for the treatment of primary myelofibrosis. While the patient is taking this medication, what will the nurse monitor to determine effectiveness? Leukocyte and platelet count Blood urea nitrogen (BUN) and creatinine levels Aspartate aminotransferase (AST) and alanine transaminase (ALT) levels Hemoglobin and hematocrit

Leukocyte and platelet count Explanation: Hydroxyurea is often used in patients with primary myelofibrosis to control high leukocyte and platelet counts and to reduce the size of the spleen.

A 63-year-old woman has been diagnosed with polycythemia vera (PV) after undergoing a series of diagnostic tests. When the woman's nurse is providing health education, what subject should the nurse prioritize? - Maintenance of long-term vascular access device - Lifestyle modifications and techniques for preventing thromboembolism - Strategies for managing activity - Nutritional modifications necessary for maintaining a low-iron diet

Lifestyle modifications and techniques for preventing thromboembolism The increased blood volume and viscosity that are the hallmarks of PV create a significant risk of thromboembolism. A vascular access device is not necessary for the treatment of PV, and a low-iron diet does not resolve the disease. Patients may experience fatigue, but this risk is superseded by that of thromboembolism.

A nurse prepares teaching for a group of clients with chronic myeloid leukemia (CML). When planning the teaching on medication adherence, which factors associated with lower oral therapy adherence will the nurse keep in mind? Select all that apply.

Living alone Low socioeconomic status Not participating in a clinical trial Taking medication independent of meals

Which term refers to a form of white blood cell involved in immune response? Granulocyte Lymphocyte Spherocyte Thrombocyte

Lymphocyte Explanation: Both B and T lymphocytes respond to exposure to antigens. Granulocytes include basophils, neutrophils, and eosinophils. A spherocyte is a red blood cell without central pallor, seen with hemolysis. A thrombocyte is a platelet.

Which term refers to a form of white blood cell involved in immune response? - Spherocyte - Thrombocyte - Lymphocyte - Granulocyte

Lymphocyte Both B and T lymphocytes respond to exposure to antigens. Granulocytes include basophils, neutrophils, and eosinophils. A spherocyte is a red blood cell without central pallor, seen with hemolysis. A thrombocyte is a platelet.

A pt has completed induction therapy and has diarrhea and severe mucositis. What is the appropriate nursing goal?

Maintain nutrition

A client has completed induction therapy and has diarrhea and severe mucositis. What is the appropriate nursing goal? Address issues of negative body image. Place the client in reverse isolation. Administer pain medication. Maintain nutrition.

Maintain nutrition. Explanation: Maintaining nutrition is the most important goal after induction therapy because the client experiences severe diarrhea and can easily become nutritionally deficient and develop fluid and electrolyte imbalance. The client is most likely not in pain at this point, and this is an intervention, not a goal.

The nurse cares for an older adult client with unprovoked back pain and increased serum protein. Which hematologic neoplasm does the nurse suspect the client has?

Multiple myeloma Any older adult with unprovoked or unexplained back pain and increased protein in the serum should be assessed for multiple myeloma. Bone pain occurs because of bone breakdown and the malignant cells of multiple myeloma increase the serum protein levels.

The nurse cares for an older adult client with unprovoked back pain and increased serum protein. Which hematologic neoplasm does the nurse suspect the client has? Chronic myeloid leukemia Multiple myeloma Hodgkin lymphoma Non-Hodgkin lymphoma

Multiple myeloma Explanation: Any older adult with unprovoked or unexplained back pain and increased protein in the serum should be assessed for multiple myeloma. Bone pain occurs because of bone breakdown and the malignant cells of multiple myeloma increase the serum protein levels.

A client was admitted to the hospital with a pathologic pelvic fracture. The client informs the nurse that he has been having a strange pain in the pelvic area for a couple of weeks that was getting worse with activity prior to the fracture. What does the nurse suspect may be occurring based on these symptoms? - Multiple myeloma - Leukemia - Hemolytic anemia - Polycythemia vera

Multiple myeloma The first symptom usually is vague pain in the pelvis, spine, or ribs. As the disease progresses, the pain becomes more severe and localized. The pain intensifies with activity and is relieved by rest. When tumors replace bone marrow, pathologic fractures develop. Hemolytic anemia does not result in pathologic fractures nor does polycythemia vera or leukemia.

A client with acute myeloid leukemia has a fever. What pathophysiological process does the nurse recognize is the cause of the client's fever?

Neutropenia

A client with acute myeloid leukemia has a fever. What pathophysiological process does the nurse recognize is the cause of the client's fever? Pancytopenia Thrombocytopenia Anemia Neutropenia

Neutropenia Explanation: Fever and infection result from a decrease in neutrophils (neutropenia). Decreased red blood cells (anemia) cause weakness, fatigue, dyspnea on exertion, and pallor in AML. Pancytopenia, an overall decrease in all blood components, is not cause of fever in clients with AML. Decreased platelet count (thrombocytopenia) causes petechiae and bruising in AML.

What assessment findingbest indicates that the client has recovered from induction therapy? Neutrophil and platelet counts within normal limits Vital signs within normal ranges No evidence of edema Absence of bone pain

Neutrophil and platelet counts within normal limits Explanation: Recovery from induction therapy is indicated when the neutrophil and platelet counts have returned to normal and any infection has resolved. Stable vital signs, lack of edema, and absence of pain are not indicative of recovery from induction therapy.

The hospitalized client is experiencing gastrointestinal bleeding with a platelets at 9,000/mm³. The client is receiving prednisone and azathioprine. What action will the nurse take? Use contact precautions with this client. Perform a neurologic assessment with vital signs. Request a prescription of diphenoxylate and atropine for loose stools. Teach the client to vigorously floss the teeth to prevent infections.

Perform a neurologic assessment with vital signs. Explanation: With platelets less than 10,000/mm³ there is a risk for spontaneous bleeding, including within the cranial vault. The nurse performs a neurologic examination to assess for this possibility. Though the client is receiving immunosuppressants, it is not necessary to use contact precautions with this client. Contact precautions are used with clients who have known or suspected transmittable illnesses. Diphenoxylate and atropine can cause constipation and inhibit accurate assessment of the client's gastrointestinal bleeding. If the client strains when having a bowel movement, the client could bleed even more. The client is not to floss vigorously; doing so can cause bleeding.

A nurse is caring for an asymptomatic client with acute myelogenous leukemia. The client has a total white blood cell (WBC) count of 0 ?l, a platelet count of 3,000 mm2, and a hemoglobin level of 9 mg/dl. He has a single lumen central venous catheter in place and the physician has ordered the nurse to administer imipenem cilastatin (Primaxin) 500 mg every 8 hours, transfuse 1 unit packed red blood cells (RBCs), give amphotericin B (Fungizone) 40 mg I.V. over 4 hours, and transfuse 2 pheresis units of platelets. In what order should the nurse infuse these medications and blood products?

Platelets, imipenem cilastatin, amphotericin B, packed RBCs Although the client is currently asymptomatic, a platelet count of 3,000 mm2 puts him at risk for spontaneous hemorrhage, the most immediate and serious risk he faces. A WBC count of 0 clearly indicates neutropenia; the client needs an antibiotic and antifungal therapy to prevent infection. Although the client is anemic, he's currently asymptomatic. The absence of clinical manifestations makes his need for a transfusion less urgent.

The nursing instructor is discussing disorders of the hematopoietic system with the pre-nursing pathophysiology class. What disease would the instructor list with a primary characteristic of erythrocytosis? Polycythemia vera Sickle cell disease Aplastic anemia Pernicious anemia

Polycythemia vera Explanation: Polycythemia vera is associated with a rapid proliferation of blood cells produced by the bone marrow. In sickle cell disease, HbS causes RBCs to assume a sickled shape under hypoxic conditions. Aplastic anemia has a deficiency of erythrocytes. The other options do not have the characteristics of erythrocytosis.

Which statement best describes the function of stem cells in the bone marrow? They are active against hypersensitivity reactions. They defend against bacterial infection. They produce all blood cells. They produce antibodies against foreign antigens.

They produce all blood cells. Explanation: All blood cells are produced from undifferentiated precursors called pluripotent stem cells in the bone marrow. Other cells produced from the pluripotent stem cells help defend against bacterial infection, produce antibodies against foreign antigens, and are active against hypersensitivity reactions.

The nurse is aware that chronic lymphocytic leukemia (CLL), a common malignancy in those older than 60, has an early stage and a late stage. The nurse assesses a patient for late stage CLL by looking for:

Thrombocytopenia. Anemia and thrombocytopenia are late-stage indicators of CLL. The others are early-stage signs.

The nurse caring for an older adult with a diagnosis of leukemia would encourage the client to use an electric razor. Why? - The client is at risk for spontaneous and uncontrolled bleeding. - Trauma and microabrasions may contribute to anemia. - Fragile tissues and altered clotting mechanisms may result in hemorrhage. - The client is at risk for infection from microorganisms.

Trauma and microabrasions may contribute to anemia. In a client with leukemia who is at risk for hemorrhage, the nurse handles the client gently when assisting and encourages the client to use electric razors. Trauma and microabrasions from razors may contribute to anemia from bleeding. Fragile tissues and altered clotting mechanisms may result in hemorrhage even after minor trauma. Therefore, the nurse inspects the skin for signs of bruising and petechiae and reports melena, hematuria, or epistaxis (nosebleeds). The risks for spontaneous and uncontrolled bleeding or infection from microorganisms are not addressed by the use of electric razors.

A patient is being evaluated for a diagnosis of chronic myeloid leukemia (CML). The nurse understands that a diagnostic indicator is: a) A leukocyte count >100,000/mm3. b) Lymphadenopathy. c) Increased number of blast cells. d) An enlarged liver.

a) A leukocyte count >100,000/mm3. Although there is an increase in the production of blast cells, and the patient may have an enlarged liver and tender spleen, it is the high leukocyte count that is diagnostic. Lymphadenopathy is rare.

A patient with polycythemia vera is complaining of severe itching. What triggers does the nurse know can cause this distressing symptom? (Select all that apply.) a) Alcohol consumption b) Allergic reaction to the red blood cell increase c) Temperature change d) Exposure to water of any temperature e) Aspirin

a) Alcohol consumption c) Temperature change d) Exposure to water of any temperature Pruritus is very common, occurring in up to 70% of patients with polycythemia vera (Saini, Patnaik & Tefferi, 2010) and is one of the most distressing symptoms of this disease. It is triggered by contact with temperature change, alcohol consumption, or, more typically, exposure to water of any temperature but seems to be worse with exposure to hot water.

A nurse has established for a client the nursing diagnosis of risk for infection. Which of the following interventions would the nurse include in the plan of care for this client? Select all answers that apply. a) Encourage the client to take deep breaths every 4 hours while awake. b) Place fresh flowers on a shelf on the opposite wall from the client. c) Auscultate lung sounds every shift and prn. d) Assess skin and mucus membranes every shift. e) Provide oral hygiene once daily.

a) Encourage the client to take deep breaths every 4 hours while awake. c) Auscultate lung sounds every shift and prn. d) Assess skin and mucus membranes every shift. Interventions for risk for infection include assessing skin and mucus membranes every shift, auscultating lung sounds every shift and prn, and encouraging deep breaths every 4 hours while the client is awake. No fresh flowers are allowed in the room because of germs found in stagnant water. Oral hygiene should be provided after meals and every 4 hours while the client is awake.

A patient with leukemia has developed a cough and increased fatigue. What is the primary nursing intervention? a) Evaluate the patient for potential infection. b) Place a cooling blanket on the patient. c) Administer an antitussive. d) Medicate the patient for pain.

a) Evaluate the patient for potential infection. Explanation: The patient with leukemia has a lack of mature and normal granulocytes for fighting infection. For this reason, the patient is susceptible to infection. The primary nursing intervention is to evaluate the patient for potential infection if he or she has a cough and increased fatigue. Administering an antitussive would not be appropriate before determining the cause of the cough. A cooling blanket would not be needed if the patient does not have a fever. Medicating the patient for pain would come after the assessment phase.

A patient with polycythemia vera has a high red blood cell (RBC) count and is at risk for the development of thrombosis. What treatment is important to reduce blood viscosity and to deplete the patient's iron stores? a) Phlebotomy b) Blood transfusions c) Radiation d) Chelation therapy

a) Phlebotomy The objective of management is to reduce the high RBC count and reduce the risk of thrombosis. Phlebotomy is an important part of therapy (Fig. 34-5). It involves removing enough blood (initially 500 mL once or twice weekly) to reduce blood viscosity and to deplete the patient's iron stores, thereby rendering the patient iron deficient and consequently unable to continue to manufacture hemoglobin excessively.

A nurse is caring for an asymptomatic client with acute myelogenous leukemia. The client has a total white blood cell (WBC) count of 0 ?l, a platelet count of 3,000 mm2, and a hemoglobin level of 9 mg/dl. He has a single lumen central venous catheter in place and the physician has ordered the nurse to administer imipenem cilastatin (Primaxin) 500 mg every 8 hours, transfuse 1 unit packed red blood cells (RBCs), give amphotericin B (Fungizone) 40 mg I.V. over 4 hours, and transfuse 2 pheresis units of platelets. In what order should the nurse infuse these medications and blood products? a) Platelets, imipenem cilastatin, amphotericin B, packed RBCs b) Packed RBCs, platelets, imipenem cilastatin, amphotericin B c) Amphotericin B, imipenem cilastatin, platelets, packed RBCs d) Packed RBCs, amphotericin B, imipenem cilastatin, platelets

a) Platelets, imipenem cilastatin, amphotericin B, packed RBCs Although the client is currently asymptomatic, a platelet count of 3,000 mm2 puts him at risk for spontaneous hemorrhage, the most immediate and serious risk he faces. A WBC count of 0 clearly indicates neutropenia; the client needs an antibiotic and antifungal therapy to prevent infection. Although the client is anemic, he's currently asymptomatic. The absence of clinical manifestations makes his need for a transfusion less urgent.

Which of the following is the hallmark of polycythemia vera (PV)? a) Splenomegaly b) Headache c) Ruddy complexion d) Blurred vision

a) Splenomegaly Splenomegaly is the hallmark of PV. Patients typically have a ruddy complexion and splenomegaly. Symptoms result from increased blood volume (headache, dizziness, tinnitus, fatigue, paresthesias, and blurred vision).

Which of the following are complications related to polycythemia vera (PV)? Select all that apply. a) Ulcers b) CVA c) MI d) Splenomegaly e) Hematuria

a) Ulcers b) CVA c) MI e) Hematuria Patients with PV are at increased risk for thromboses resulting in a CVA or myocardial infarction. Bleeding can be significant and can occur in the form of nosebleeds, ulcers, frank gastrointestinal bleeding, and intracranial hemorrhage. Splenomegaly is a clinical manifestation of PV, not a complication.

Which assessment findings support the client's diagnosis of AML (acute myeloid leukemia)? Select all that apply. a) Weakness and fatigue b) Enlarged heart c) Enlarged lymph nodes d) Bone pain e) Petechiae

a) Weakness and fatigue c) Enlarged lymph nodes d) Bone pain e) Petechiae Clients with AML may present with petechiae, enlarged lymph nodes, weakness, fatigue, and bone pain. An enlarged heart is not a typical finding with this disorder.

Which patient assessed by the nurse is most likely to develop myelodysplastic syndrome (MDS)? a) A 24-year-old female taking oral contraceptives b) A 72-year-old patient with a history of cancer c) A 40-year-old patient with a history of hypertension d) A 52-year-old patient with acute kidney injury

b) A 72-year-old patient with a history of cancer Primary MDS tends to be a disease of people older than 70 years. Because the initial findings are so subtle, the disease may not be diagnosed until later in the illness trajectory, if at all. Thus, the actual incidence of MDS is not known.

Following bone marrow aspiration of a 19-year-old client, analysis reveals more than 20% immature blast cells. Platelet counts are 9000/mm³. What nursing interventions should the nurse employ for the care of this client? Select all answers that apply. a) Recommend taking ibuprofen for mild aches and pains. b) Administer prescribed docusate (Colace) daily. c) Assess for mental state changes. d) Apply pressure to venipuncture sites for 1 to 2 minutes. e) Discuss the withholding of oral contraceptives.

b) Administer prescribed docusate (Colace) daily. c) Assess for mental state changes. The client has leukemia with immature blast cells and an extremely low platelet count. The client is at increased risk for bleeding. Interventions that would address bleeding include assessing for mental status changes (because bleeding could occur in the brain) and administering stool softeners to prevent constipation (which would increase the risk of bleeding from the rectum). Oral contraceptives would be administered to induce amenorrhea. Ibuprofen would be avoided because this medication inhibits platelet function. The nurse is to apply pressure to venipuncture sites for 5 minutes.

A client is receiving chemotherapy for acute myeloid leukemia and has poor nutritional intake. The first action of the nurse is to a) Provide mouth care before each meal. b) Ask, "Are you experiencing nausea?" c) Caution the client to chew carefully after administration of the prescribed lidocaine (Xylocaine Viscous). d) Provide nutritional supplements in addition to a diet that has a soft texture and moderate temperature.

b) Ask, "Are you experiencing nausea?" All these options are things the nurse can do to assist the client to obtain better nutrition. The nurse first needs to assess the reason for poor nutritional intake. It could be because of nausea, in which case the nurse would implement interventions to address the client's nausea.

The nurse practitioner suspects that a patient has multiple myeloma based on his major presenting symptom and the analysis of his laboratory results. Select the classic symptom for this disease. a) Gradual muscle paralysis b) Bone pain in the back of the ribs c) Debilitating fatigue d) Severe thrombocytopenia

b) Bone pain in the back of the ribs Although patients can have asymptomatic bone involvement, the most common presenting symptom of multiple myeloma is bone pain, usually in the back or ribs. Unlike arthritic pain, the bone pain associated with myeloma increases with movement and decreases with rest; patients may report that they have less pain on awakening but the pain intensity increases during the day.

A client is receiving radiation therapy for lesions in the abdomen from non-Hodgkin's lymphoma. Because of the effects of the radiation treatments, the nurse now assesses for a) Hair loss b) Diarrheal stools c) Adventitous lung sounds d) Laryngeal edema

b) Diarrheal stools Side effects of radiation therapy are limited to the area being irradiated. Clients who have abdominal radiation therapy may experience diarrhoea. If the lesions were in the upper chest, then the client may experience adventitious lung sounds or laryngeal oedema as side effects. Hair loss is associated more with chemotherapy than radiation therapy.

The nurse is assessing several patients. Which patient does the nurse determine is most likely to have Hodgkin lymphoma? a) The patent with painful lymph nodes under the arm. b) The patient with enlarged lymph nodes in the neck. c) The patient with painful lymph nodes in the groin. d) The patient with a painful sore throat.

b) The patient with enlarged lymph nodes in the neck. Explanation: Lymph node enlargement in Hodgkin lymphoma is not painful. The patient with enlarged lymph nodes in the neck is most likely to have Hodgkin lymphoma if the enlarged nodes are painless. Sore throat is not a sign for this disorder.

Which of the following nursing interventions should be incorporated into the plan of care to manage the delayed clotting process in a patient with leukemia? a) Eliminate direct contact with others who are infectious. b) Implement neutropenic precautions. c) Apply prolonged pressure to needle sites or other sources of external bleeding. d) Apply prolonged pressure to needle sites or other sources of external bleeding.

c) Apply prolonged pressure to needle sites or other sources of external bleeding. Explanation: For a patient with leukemia, the nurse should apply prolonged pressure to needle sites or other sources of external bleeding. Reduced platelet production results in a delayed clotting process and increases the potential for hemorrhage. Implementing neutropenic precautions and eliminating direct contact with others are interventions to address the risk for infection.

A patient with AML has pale mucous membranes and bruises on his legs. What is the primary nursing intervention? a) Assess the patient's skin. b) Check the patient's history. c) Assess the patient's hemoglobin and platelets. d) Assess the patient's pulses and blood pressure.

c) Assess the patient's hemoglobin and platelets. Explanation: Patients with AML may develop pallor from anemia and bleeding tendencies from low platelet counts. Assessing the patient's hemoglobin and platelets will help to determine if this is the cause of the symptoms. This would be the priority above assessing pulses, blood pressure, history, or skin.

A patient with AML has pale mucous membranes and bruises on his legs. What is the primary nursing intervention? a) Assess the patient's pulses and blood pressure. b) Check the patient's history. c) Assess the patient's hemoglobin and platelets. d) Assess the patient's skin.

c) Assess the patient's hemoglobin and platelets. Patients with AML may develop pallor from anemia and bleeding tendencies from low platelet counts. Assessing the patient's hemoglobin and platelets will help to determine if this is the cause of the symptoms. This would be the priority above assessing pulses, blood pressure, history, or skin.

The hospitalized client is experiencing gastrointestinal bleeding. Laboratory test results show that the client's platelets are 9000/mm³. The client is receiving prednisone and azathioprine (Imuran). The nurse a) Teaches the client to vigorously floss the teeth to prevent infections b) Requests a prescription of diphenoxylate/atropine (Lomotil) for loose stools c) Performs a neurologic assessment with vital signs d) Uses contact precautions with this client

c) Performs a neurologic assessment with vital signs With platelets less than 10,000/mm³ there is a risk for spontaneous bleeding, including within the cranial vault. The nurse performs a neurologic examination to assess for this possibility. Though the client is receiving immunosuppressants, it is not necessary to use contact precautions with this client. Contact precautions are used with clients who have known or suspected transmittable illnesses. Diphenoxylate/atropine can cause constipation and inhibit accurate assessment of the client's gastrointestinal bleeding. If the client strains when having a bowel movement, the client could bleed even more. The client is not to floss vigorously; doing so can cause bleeding.

A nurse is caring for a client with multiple myeloma. Which laboratory value is the nurse most likely to see? a) Hypermagnesemia b) Hypernatremia c) Hyperkalemia d) Hypercalcemia

d) Hypercalcemia Calcium is released when bone is destroyed, causing hypercalcemia. Multiple myeloma doesn't affect potassium, sodium, or magnesium levels.

A patient with suspected multiple myeloma is complaining of pain in the back. What is the priority nursing action? a) Have the patient rest. b) Have the patient lie on a hard surface. c) Encourage ambulation. d) Send the patient for x-ray study of the spine.

d) Send the patient for x-ray study of the spine. Explanation: The patient with myeloma can have bone pain, especially in the back and ribs. The pain will decrease with rest and increase with activity. Lying on a hard surface will not relieve the pain. The priority action is to make certain the patient does not have a fracture of the spine, as the bone destruction in this disease is sufficiently severe to cause vertebral collapse.

Clinical assessment of a patient with AML includes observing for signs of infection, the major cause of death for AML. The nurse should assess for indicators of: a) Neutropenia. b) Thrombocytopenia. c) Bone marrow expansion. d) Splenomegaly.

d) Splenomegaly. Acute myeloid leukemia starts inside the bone marrow and prevents the formation of white blood cells. A bone marrow analysis that shows greater than 30% of immature blast cells is indicative of an AML diagnosis.

lysis

destruction of cells

Splenomegaly

enlargement of the spleen

Angiogenesis

formation of new blood vessels

Neutrophil

fully mature WBC capable of phagocytosis; primary defense against bacterial infection

Granulocyte

granulated WBC (neutrophil, eosinophil, basophil); term sometimes used as synonymous with neutrophil

For a client with Hodgkin lymphoma, who is at a risk for ineffective airway clearance and impaired gas exchange, the nurse places the client in a high Fowler's position to

increase lung expansion.

For a pt with Hodgkin disease, who is at a risk for an ineffective airway clearance and an impaired gas exchange, the nurse places the pt in a high fowler's position to do what?

increase the lung expansion

A client has been diagnosed with multiple myeloma. Which of the following lab values should the nurse expect to find in a client with multiple myeloma?

increased urinary protein

Two primary complications seen in acute myeloid leukemia (AML), bleeding and __________, are the major causes of death for these patients.

infection

Erythrocyte sedimentation rate (ESR)

laboratory test that measures the rate of settling of red blood cells (RBCs); elevation is indicative of inflammation; also called the sed rate

Monocyte

large WBC that becomes a macrophage when it leaves the circulation and moves into body tissues

Leukopenia

less-than-normal amount of WBCs in circulation

Neutropenia

lower-than-normal number of neutrophils

Thrombocytopenia

lower-than-normal platelet count

A nurse is assessing a client with multiple myeloma. The nurse should keep in mind that clients with multiple myeloma are at risk for: - hypoxemia. - pathologic bone fractures. - chronic liver failure. - acute heart failure.

pathologic bone fractures. Clients with multiple myeloma are at risk for pathologic bone fractures secondary to diffuse osteoporosis and osteolytic lesions. Also, clients are at risk for renal failure secondary to myeloma proteins by causing renal tubular obstruction. Liver failure and heart failure aren't usually sequelae of multiple myeloma. Hypoxemia isn't usually related to multiple myeloma.

The nurse is discussing disorders of the hematopoietic system when a client asked about erythrocytosis. What disease will the nurse mention with a primary characteristic of erythrocytosis? polycythemia vera sickle cell disease aplastic anemia pernicious anemia

polycythemia vera Explanation: Polycythemia vera is associated with a rapid proliferation of blood cells produced by the bone marrow. In sickle cell disease, HbS causes RBCs to assume a sickled shape under hypoxic conditions. Aplastic anemia has a deficiency of erythrocytes. Sickle cell disease and the anemias do not have the characteristics of erythrocytosis.

Phagocytosis

process of cellular ingestion and digestion of foreign bodies

Clone

proliferation from same cell of origin so that descendent cells are identical to the cell of origin

The nurse is assessing a client admitted with a deep vein thrombosis with an elevated red blood cell count. The admitting diagnosis is polycythemia vera. What is the hallmark clinical sign of PV?

splenomegaly

The nurse is assessing a client admitted with a deep vein thrombosis with an elevated red blood cell count. The admitting diagnosis is polycythemia vera. What is the hallmark clinical sign of PV? splenomegaly weight gain peripheral edema pale body color

splenomegaly Explanation: Splenomegaly is the hallmark of PV. Patients typically have a ruddy complexion and splenomegaly. Symptoms result from increased blood volume (headache, dizziness, tinnitus, fatigue,weight loss, paresthesias, and blurred vision). Edema, pale body color, and weight gain are not classic symptoms of PV.

The nurse is caring for a pt who will begin taking long-term biophosphate therapy. Why is it important for the nurse to encourage the pt to receive a thorough evaluation of dentition, including panoramic dental x-ray?

the pt can develop osteonecrosis of the jaw

Lymphocyte

type of WBC involved in immune functions

Leukemia

uncontrolled proliferation of WBCs, often immature

A nurse is assessing a client with multiple myeloma. Due to this condition, what will this client be at risk for?

pathologic bone fractures.

A nurse is assessing a client with multiple myeloma. The nurse should keep in mind that clients with multiple myeloma are at risk for...

pathologic bone fractures.

A patient is taking hydroxyurea for the treatment of primary myelofibrosis. While the patient is taking this medication, what will the nurse monitor to determine effectiveness?

Leukocyte and platelet count Hydroxyurea is often used in patients with primary myelofibrosis to control high leukocyte and platelet counts and to reduce the size of the spleen.

What assessment findings support the client's diagnosis of AML (acute myeloid leukemia)?

Petechiae, enlarged lymph nodes, weakness and fatigue, and bone pain

A client with multiple myeloma presents to the emergency department and reports excessive thirst and constipation. Family members report that the client has been confused for the last day. Which laboratory value is most likely responsible for this client's symptoms?

Serum calcium level 13.8 mg/dl

Clinical assessment of a pt with AML includes observing for signs of infection, the major cause of death for AML. The nurse should assess for indicators of:

Splenomegaly

A pt presents with peripheral neuropahty and hypothesia of the ft. What is the best nursing intervention?

assess for signs of injury

An older adult with a chief complaint of ______________ and an elevated total protein level should be evaluated for multiple myeloma.

back pain

Which statement indicates the client understands teaching about induction therapy for leukemia? - "I will start slowly with medication treatment." - "I will need to come every week for treatment." - "I know I can never be cured." - "I will be in the hospital for several weeks."

"I will be in the hospital for several weeks." Induction therapy involves high doses of several medications and the client is usually admitted to the hospital for several weeks. The treatment is started quickly and the goal is to cure or put the disease into remission.

Which statement indicates the client understands teaching about induction therapy for leukemia? "I will start slowly with medication treatment." "I will need to come every week for treatment." "I will be in the hospital for several weeks." "I know I can never be cured."

"I will be in the hospital for several weeks." Explanation: Induction therapy involves high doses of several medications and the client is usually admitted to the hospital for several weeks. The treatment is started quickly and the goal is to cure or put the disease into remission.

The nurse is teaching a client about the development of leukemia. What statement should be included in the teaching plan?

"Chronic leukemia develops slowly."

The nurse is teaching a client about the development of leukemia. What statement should be included in the teaching plan? "Acute leukemia develops slowly." "Chronic leukemia develops slowly." "In chronic leukemia, the minority of leukocytes are mature." "In acute leukemia there are not many undifferentiated cells."

"Chronic leukemia develops slowly." Explanation: Chronic leukemia develops slowly, and the majority of leukocytes produced are mature. Acute leukemia develops quickly and the majority of leukocytes are undifferentiated cells.

The nurse is teaching the client about consolidation. What statement should be included in the teaching plan? "Consolidation therapy is administered to reduce the chance of leukemia recurrence." "Consolidation occurs as a side effect of chemotherapy." "Consolidation of the lungs is an expected effect of induction therapy." "Consolidation is the term used when a client does not tolerate chemotherapy."

"Consolidation therapy is administered to reduce the chance of leukemia recurrence." Explanation: Consolidation therapy is administered to eliminate residual leukemia cells that are not clinically detectable and reduce the chance for recurrence. It is also termed postremission therapy. It is not a side effect of chemotherapy, but the administration of chemotherapy.

The nurse is teaching a client with acute lymphocytic leukemia (ALL) about therapy. What statement should be included in the plan of care?

"Intrathecal chemotherapy is used primarily as preventive therapy."

The nurse is teaching a client with acute lymphocytic leukemia (ALL) about therapy. What statement should be included in the plan of care?

"Intrathecal chemotherapy is used primarily as preventive therapy." Intrathecal chemotherapy is a key part of the treatment plan to prevent invasion of the central nervous system. The therapy uses multiple drugs, with many side effects. The goal of therapy is remission.

A 71-year-old woman with a history of rheumatoid arthritis and chronic heart failure has been admitted to the hospital for the treatment of a suspected upper gastrointestinal bleed. When performing an assessment of this patient, which of the following questions most directly addresses a likely cause of the woman's bleeding disorder? - "Did either of your parents or siblings have problems with bleeding?" - "How closely do you tend to monitor your blood pressure when you're at home?" - "Has your doctor prescribed a water pill for your heart failure?" - "Do you ever take aspirin to treat the pain of your arthritis?"

"Do you ever take aspirin to treat the pain of your arthritis?" An important functional platelet disorder is that induced by aspirin. Even small amounts of aspirin reduce normal platelet aggregation, and the prolonged bleeding time lasts for several days after aspirin ingestion. Diuretics, hypertension, and family history are not central parameters in the assessment of a bleeding disorder.

A client with leukemia is being discharged from the hospital to hospice care. Which statement by the client indicates the client has not achieved the goal for the nursing diagnosis Spiritual Distress? - "I do not understand why this happened to me." - "I know I am going to die. I want to say good-bye to my family." - "I am going to call my clergy to pray with me." - "I have resources within myself that I can depend on."

"I do not understand why this happened to me." The statement "I do not understand why this happened to me" indicates that the client is not accepting of the consequences of his health problems and impending death. The other statements indicate the client has plans that would result in spiritual well-being or harmony.

Which statement indicates the client understands teaching about induction therapy for leukemia?

"I will be in the hospital for several weeks."

The nurse is teaching a client with acute lymphocytic leukemia (ALL) about therapy. What statement should be included in the plan of care? "Treatment is simple and consists of single-drug therapy." "Intrathecal chemotherapy is used primarily as preventive therapy." "The goal of therapy is palliation." "Side effects are rare with therapy."

"Intrathecal chemotherapy is used primarily as preventive therapy." Explanation: Intrathecal chemotherapy is a key part of the treatment plan to prevent invasion of the central nervous system. The therapy uses multiple drugs, with many side effects. The goal of therapy is remission.

A nurse is teaching a client with multiple myeloma about the therapeutic benefits of radiation therapy. Which statements will the nurse include in the teaching? Select all that apply.

"It helps to strengthen the bone." "It helps to decrease bone pain."

A nurse is providing teaching to a client who will undergo chemotherapy and radiation prior to hematopoietic stem cell transplantation (HSCT) for acute myeloid leukemia (AML). What statement will the nurse use to describe the purpose of the chemotherapy and radiation?

"These therapies destroy the ability of your body to produce blood cells inside your bone marrow." The treatment goal of chemotherapy and radiation therapy is the destruction of hematopoietic function of the client's bone marrow. The client is then "rescued" with the infusion of the donor stem cells to reinitiate blood cell production. AML is a cancer of the blood and does not have a mass effect/tumor that other cancers may cause. Also, these therapies are not used to decrease a client's pain or to decrease the risk of allergic reaction.

A nurse cares for an adult client with chronic lymphocytic leukemia (CLL). Which statements regarding the disease will the nurse include in the teaching?

"This type of leukemia primarily impacts older adults." "This type of leukemia is rarely seen in certain ethnicities." Chronic lymphocytic leukemia (CLL) is a common malignancy of older adults and primarily impacts older adults and has a strong familial predisposition. This type of leukemia rarely impacts Native Americans and infrequently individuals of Asian descent. While many clients will have a normal life expectancy, others will have a very short life expectancy due to the aggressive nature of the disease.

A patient with polycythemia vera is complaining of severe itching. What triggers does the nurse know can cause this distressing symptom? (Select all that apply.)

-Temperature change -Alcohol consumption -Exposure to water of any temperature Pruritus is very common, occurring in up to 70% of patients with polycythemia vera (Saini, Patnaik & Tefferi, 2010) and is one of the most distressing symptoms of this disease. It is triggered by contact with temperature change, alcohol consumption, or, more typically, exposure to water of any temperature but seems to be worse with exposure to hot water.

The nurse is caring for a client with multiple myeloma. Why would it be important to assess this client for fractures? A.) Osteopathic tumors destroy bone causing fractures. B.) Osteoclasts break down bone cells so pathologic fractures occur. C.) Osteolytic activating factor weakens bones producing fractures. D.) Osteosarcomas form producing pathologic fractures.

Answer: B.) Osteoclasts break down bone cells so pathologic fractures occur.

A client is being evaluated for a diagnosis of chronic myeloid leukemia (CML). What diagnostic indicator will the nurse assess? An enlarged liver A leukocyte count >100,000/mm3 Lymphadenopathy Increased number of blast cells

A leukocyte count >100,000/mm3 Explanation: Although there is an increase in the production of blast cells and the client may have an enlarged liver and tender spleen, it is the high leukocyte count that is diagnostic. Lymphadenopathy is rare.

Which patient assessed by the nurse is most likely to develop myelodysplastic syndrome (MDS)?

A 72-year-old patient with a history of cancer

Which patient assessed by the nurse is most likely to develop myelodysplastic syndrome (MDS)?

A 72-year-old patient with a history of cancer Primary MDS tends to be a disease of people older than 70 years. Because the initial findings are so subtle, the disease may not be diagnosed until later in the illness trajectory, if at all. Thus, the actual incidence of MDS is not known.

Which patient assessed by the nurse is most likely to develop myelodysplastic syndrome (MDS)? A 24-year-old female taking oral contraceptives A 40-year-old patient with a history of hypertension A 52-year-old patient with acute kidney injury A 72-year-old patient with a history of cancer

A 72-year-old patient with a history of cancer Explanation: Primary MDS tends to be a disease of people older than 70 years. Because the initial findings are so subtle, the disease may not be diagnosed until later in the illness trajectory, if at all. Thus, the actual incidence of MDS is not known.

What type of pt is most likely to develop myelodyplastic syndrome?

A 72-yr-old pt with a hx of cancer

The clinic nurse is caring for a client diagnosed with leukopenia. What does the nurse know the client has?

A general reduction in all WBCs

The clinic nurse is caring for a client diagnosed with leukopenia. What does the nurse know this client has?

A general reduction in all white blood cells

The clinic nurse is caring for a client diagnosed with leukopenia. What does the nurse know this client has? Too many erythrocytes A decrease in granulocytes A general reduction in all white blood cells A general reduction in neutrophils and basophils

A general reduction in all white blood cells Explanation: Leukopenia is a general reduction in all WBCs. Leukopenia does not have anything to do with erythrocytes.

A client is receiving chemotherapy for acute myeloid leukemia and has poor nutritional intake. The first action of the nurse is to

Ask, "Are you experiencing nausea?" All these options are things the nurse can do to assist the client to obtain better nutrition. The nurse first needs to assess the reason for poor nutritional intake. It could be because of nausea, in which case the nurse would implement interventions to address the client's nausea.

The nurse practitioner suspects that a pt has multiple myeloma based on his major presenting symptom and the analysis of his lab results. What's the classic symptom of the disease?

Bone pain in the back of the ribs

A nurse assesses a client who has been diagnosed with DIC. Which indicators are consistent with this diagnosis? Select all that apply. - Cyanosis in the extremities - Capillary fill time <3 seconds - Dyspnea and hypoxia - Increased breath sounds - Increased blood urea nitrogen (BUN) and creatinine - Polyuria

Answer: - Cyanosis in the extremities - Dyspnea and hypoxia - Increased blood urea nitrogen (BUN) and creatinine Rationale: Urine output would be decreased in DIC, and capillary fill time would be more than 3 seconds; breath sounds would be decreased.

The nurse suspects that a client has multiple myeloma based on the client's major presenting symptom and the analysis of laboratory results. What classic symptom for multiple myeloma does the nurse assess for?

Bone pain in the back of the ribs

Your client has just been prescribed oral iron. Why would you advise this client to avoid taking their medication with coffee, tea, eggs, or milk? - Untoward reactions may occur. - Coffee, tea, eggs, and milk interact with oral iron. - Grand mal seizures may result. - Absorption of iron will decrease.

Absorption of iron will decrease. When a client takes the drug with coffee, tea, eggs, or milk, absorption of oral iron decreases. The use of meperidine or Demerol when treating pain in clients with sickle cell crisis may result in grand mal seizures. Antacids, tetracyclines, and vitamin C interact with oral iron.

The nurse is teaching a pt about the development fo leukemia. What statement should be included in the teaching plan?

Chronic leukemia develops slowly

Which of the following is the only curative treatment for chronic myeloid leukemia (CML)?

Allogeneic stem cell transplant Allogeneic stem cell transplantation remains the only curative treatment for CML. The efficacy of Imatinib as first-line treatment and the treatment-related mortality of stem cell transplant limits use of transplant to patients with high risk or relapsed disease, or in those patients who did not respond to therapy with TKI. Cytarabine and idarubicin are part of induction therapy for acute myeloid leukemia (AML).

Which of the following is the only curvative treatment for chronic myeloid leukemia (CML)?

Allogenic stem cell transplant

The nurse is caring for a client with acute myeloid leukemia (AML) with high uric acid levels. What medication does the nurse anticipate administering that will prevent crystallization of uric acid and stone formation?

Allopurinol

The hospitalized client is experiencing gastrointestinal bleeding with a platelets at 9,000/mm³. The client is receiving prednisone and azathioprine. What action will the nurse take? A.) Use contact precautions with this client. B.) Perform a neurologic assessment with vital signs. C.) Request a prescription of diphenoxylate and atropine for loose stools. D.) Teach the client to vigorously floss the teeth to prevent infections.

Answer: B.) Perform a neurologic assessment with vital signs. Rationale: With platelets less than 10,000/mm³ there is a risk for spontaneous bleeding, including within the cranial vault. The nurse performs a neurologic examination to assess for this possibility. Though the client is receiving immunosuppressants, it is not necessary to use contact precautions with this client. Contact precautions are used with clients who have known or suspected transmittable illnesses. Diphenoxylate and atropine can cause constipation and inhibit accurate assessment of the client's gastrointestinal bleeding. If the client strains when having a bowel movement, the client could bleed even more. The client is not to floss vigorously; doing so can cause bleeding.

A client with AML has pale mucous membranes and bruises on the legs. What is the primary nursing intervention? - Assess the client's hemoglobin and platelets. - Assess the client's pulse and blood pressure. - Check the client's history. - Assess the client's skin.

Assess the client's hemoglobin and platelets. Clients with AML may develop pallor from anemia and a tendency to bleed because of a low platelet count. Assessing the client's hemoglobin and platelets will help to determine whether this is the cause of the symptoms. This would be the priority above assessing pulses, blood pressure, history, or skin.

A pt with AML has pale mucous membranes adn bruises on his legs. What is the primary nursing intervention?

Assess the pt's hemoglobin and platelets

A pt presents with peripheral neuropathy and hypothesia of the feet. What is the best nursing intervention?

Assess the signs of injury

An elderly client is hospitalized for induction of chemotherpay to treat leukemia. The client reports fatigue to the nurse. What nursing intervention would best address the client's fatigue?

Assist the client to sit in a chair for meals

An elderly client is hospitalized for induction of chemotherapy to treat leukemia. The client reports fatigue to the nurse. What nursing intervention would best address the client's fatigue?

Assist the client to sit in a chair for meals.

The nurse practitioner suspects that a patient has multiple myeloma based on his major presenting symptom and the analysis of his laboratory results. Select the classic symptom for this disease.

Bone pain in the back of the ribs Although patients can have asymptomatic bone involvement, the most common presenting symptom of multiple myeloma is bone pain, usually in the back or ribs. Unlike arthritic pain, the bone pain associated with myeloma increases with movement and decreases with rest; patients may report that they have less pain on awakening but the pain intensity increases during the day.

The nurse suspects that a client has multiple myeloma based on the client's major presenting symptom and the analysis of laboratory results. What classic symptom for multiple myeloma does the nurse assess for? Debilitating fatigue Bone pain in the back of the ribs Gradual muscle paralysis Severe thrombocytopenia

Bone pain in the back of the ribs Explanation: Although patients can have asymptomatic bone involvement, the most common presenting symptom of multiple myeloma is bone pain, usually in the back or ribs. Unlike arthritic pain, the bone pain associated with myeloma increases with movement and decreases with rest; clients may report that they have less pain on awakening but the pain intensity increases during the day.

The nurse practitioner suspects that a patient has multiple myeloma based on his major presenting symptom and the analysis of his laboratory results. Select the classic symptom for this disease. - Severe thrombocytopenia - Bone pain in the back of the ribs - Gradual muscle paralysis - Debilitating fatigue

Bone pain in the back of the ribs Although patients can have asymptomatic bone involvement, the most common presenting symptom of multiple myeloma is bone pain, usually in the back or ribs. Unlike arthritic pain, the bone pain associated with myeloma increases with movement and decreases with rest; patients may report that they have less pain on awakening but the pain intensity increases during the day.

A client diagnosed with polycythemia vera has come into the clinic because he has developed a nighttime cough, fatigue, and shortness of breath. What complication would you suspect in this client?

Congestive heart failure

A client has multiple myeloma. Prior to starting treatment with thalidomide, what it is most important action for the nurse to take?

Contract with the client regarding birth-control methods.

A nurse assesses a client who has been diagnosed with DIC. Which indicators are consistent with this diagnosis? Select all that apply.

Cyanosis in the extremities Dyspnea and hypoxia Increased blood urea nitrogen (BUN) and creatinine

A nurse assesses a client who has been diagnosed with DIC. Which indicators are consistent with this diagnosis? Select all that apply. Cyanosis in the extremities Capillary fill time <3 seconds Dyspnea and hypoxia Increased breath sounds Increased blood urea nitrogen (BUN) and creatinine Polyuria

Cyanosis in the extremities Dyspnea and hypoxia Increased blood urea nitrogen (BUN) and creatinine Explanation: Urine output would be decreased in DIC, and capillary fill time would be more than 3 seconds; breath sounds would be decreased.

T or F- Hematopoiesis is characterized by a slow, continuous process of the formation and maturation of blood cells.

False

What type of cancer is the most common type of secondary malignancy in patients with Hodgkin's disease?

Lung Lung cancer is the most common type of secondary malignancy in patients with Hodgkin's disease, particularly following combination chemotherapy and radiation. Breast, colon, and bone are not the most common type of secondary malignancy.

A client was admitted to the hospital with a pathologic pelvic fracture. The client informs the nurse that he has been having a strange pain in the pelvic area for a couple of weeks that was getting worse with activity prior to the fracture. What does the nurse suspect may be occurring based on these symptoms?

Multiple myeloma

The nurse cares for an older adult client with unprovoked back pain and increased serum protein. Which hematologic neoplasm does the nurse suspect the client has?

Multiple myeloma

A client was admitted to the hospital with a patho pelvic fracture. The client informs the nurse that he has been having a strange pain in the pelvic area for a couple of weeks that was getting worse with activity prior to the fracture. What does the nurse suspect may be occurring based on these symptoms?

Mutiple myeloma

What assessment findingbest indicates that the client has recovered from induction therapy? - Absence of bone pain - No evidence of edema - Vital signs within normal ranges - Neutrophil and platelet counts within normal limits

Neutrophil and platelet counts within normal limits Recovery from induction therapy is indicated when the neutrophil and platelet counts have returned to normal and any infection has resolved. Stable vital signs, lack of edema, and absence of pain are not indicative of recovery from induction therapy.

Which term refers to an abnormal decrease in white blood cells, red blood cells, and platelets?

Pancytopenia

The nurse recognizes the clinical assessment of a patient with acute myeloid leukemia (AML) includes observing for signs of infection early. What nursing action will most likely help prevent infection? Monitor the client's temperature every shift. Maintain contact precautions. Encouarge increased fluid consumption. Practice vigilant handwashing.

Practice vigilant handwashing. Explanation: Infection prevention is best handled by vigilant handwashing. Monitoring the client's temperature once a shift is not often enough. The client will take precautions, but precautions are enough to prevent infections. Encouarging increased fluid consumption will not prevent infection.

A nurse is caring for a client with multiple myeloma. Which nursing intervention is most appropriate for this client?

Preventing bone injury

Which nursing intervention is most appropriate for a client with multiple myeloma?

Preventing bone injury

Which nursing intervention is most appropriate for a client with multiple myeloma?

Preventing bone injury When caring for a client with multiple myeloma, the nurse should focus on relieving pain, preventing bone injury and infection, and maintaining hydration. Monitoring respiratory status and balancing rest and activity are appropriate interventions for any client. To prevent such complications as pyelonephritis and renal calculi, the nurse should keep the client well hydrated — not restrict his fluid intake.

A nurse is caring for a client with multiple myeloma. Which nursing intervention is most appropriate for this client? Monitoring respiratory status Balancing rest and activity Restricting fluid intake Preventing bone injury

Preventing bone injury Explanation: When caring for a client with multiple myeloma, the nurse should focus on relieving pain, preventing bone injury and infection, and maintaining hydration. Monitoring respiratory status and balancing rest and activity are appropriate interventions for any client. To prevent such complications as pyelonephritis and renal calculi, the nurse should keep the client well hydrated — not restrict fluid intake.

Which nursing intervention is most appropriate for a client with multiple myeloma? - Restricting fluid intake - Preventing bone injury - Monitoring respiratory status - Balancing rest and activity

Preventing bone injury When caring for a client with multiple myeloma, the nurse should focus on relieving pain, preventing bone injury and infection, and maintaining hydration. Monitoring respiratory status and balancing rest and activity are appropriate interventions for any client. To prevent such complications as pyelonephritis and renal calculi, the nurse should keep the client well hydrated — not restrict his fluid intake.

The nurse is caring for a patient with Hodgkin lymphoma in the hospital and preparing discharge planning education. Knowing that this patient is at risk for the development of a second malignancy, what education would be beneficial to reduce the risk factors? (Select all that apply.)

Reduce exposure to excessive sunlight Smoking cessation Decrease alcohol intake

The client was admitted to the Emergency Department after an accident with a chain saw. The client is exhibiting signs and symptoms of acute hypovolemic anemia from severe blood loss. What signs and symptoms would the nurse assess for? - Reduced urine output - Malabsorption disorders - Fatigue - Postural hypotension

Reduced urine output Acute hypovolemic anemia from severe blood loss is evidenced by the signs and symptoms of hypovolemic shock, which include reduced urine output. The symptoms of chronic hypovolemic anemia include fatigue and postural hypotension. Clients with malabsorption disorders are at great risk of iron deficiency anemia.

The diagnosis of Hodgkin lymphoma is made by means of an excisional lymph node biopsy and the finding of the __________________ cell.

Reed--Sternberg

A client with multiple myeloma reports severe paresthesia in the feet. When planning care for the client, which priority nursing diagnosis will the nurse choose?

Risk for falls

A client with multiple myeloma reports severe paresthesia in the feet. When planning care for the client, which priority nursing diagnosis will the nurse choose? Acute pain Risk for falls Impaired tissue integrity Sensory-perception disturbance

Risk for falls Explanation: A client with parathesia in the feet is at risk for falls due to impaired sensation. Acute pain, impaired tissue integrity, and sensory-perception disturbance are all nursing diagnoses that are appropriate for the client; however, risk for falls is priority.

The nurse is assessing a patent with polycythemia vera. What skin assessment data would the nurse determine is a normal finding for this patient?

Ruddy complexion

The nurse is assessing a pt with polycythemia vera. What skin assessment data would the nurse determine is a normal finding for this pt?

Ruddy complexion (redish like a tomato)

Clinical assessment of a patient with AML includes observing for signs of infection, the major cause of death for AML. The nurse should assess for indicators of: - Thrombocytopenia. - Splenomegaly. - Bone marrow expansion. - Neutropenia.

Splenomegaly. Acute myeloid leukemia starts inside the bone marrow and prevents the formation of white blood cells. A bone marrow analysis that shows greater than 30% of immature blast cells is indicative of an AML diagnosis.

A client is newly diagnosed with Hodgkin lymphoma. The nurse understands that the client's treatment will be based on what concept?

Staging of disease

A client is newly diagnosed with Hodgkin lymphoma. The nurse understands that the client's treatment will be based on what concept?

Staging of disease Treatment of Hodgkin lymphoma is based on the stage of the disease, not the histology of tissue, involvement of lymph nodes, or total blood cell count.

A client is newly diagnosed with Hodgkin lymphoma. The nurse understands that the client's treatment will be based on what concept? Histology of tissue Staging of disease Involvement of lymph nodes Total blood cell count

Staging of disease Explanation: Treatment of Hodgkin lymphoma is based on the stage of the disease, not the histology of tissue, involvement of lymph nodes, or total blood cell count.

The nurse is caring for a patient who will begin taking long-term biphosphate therapy. Why is it important for the nurse to encourage the patient to receive a thorough evaluation of dentition, including panoramic dental x-rays? The patient is at risk for tooth decay. The patient will develop gingival hyperplasia. The patient can develop osteonecrosis of the jaw. The patient can develop loosening of the teeth.

The patient can develop osteonecrosis of the jaw. Explanation: Osteonecrosis of the jaw is an infrequent but serious complication that can arise in patients treated long-term with bisphosphonates; the mandible or maxilla are affected. Careful assessment for this complication should be conducted and a thorough evaluation of the patient's dentition should be performed prior to initiating bisphosphonate therapy, including panoramic dental x-rays.

The nurse is assessing several pts. Which pt does the nurse determine is most likely to have Hodgkin lymphoma?

The pt with enlarged lymph nodes in the neck

What best describes the function of stem cells in the bone marrow?

They produce all blood cells

What statement best describes the function of stem cells in the bone marrow?

They produce all blood cells

Which statement best describes the function of stem cells in the bone marrow?

They produce all blood cells.

T or F- Multiple myeloma is a malignant disease of plasma cells in bone marrow.

True

What is the rationale for the classification of leukemia?

Which bone marrow stem cell line is dysfunctional There are four general types of leukemia, classified according to the bone marrow stem cell line that is dysfunctional.

Absolute Neutrophil Count (ANC)

a calculation of the number of circulating neutrophils, derived from the total white blood cells

Red blood cell (RBC)

a cellular component of blood involved in the transport of oxygen and carbon dioxide (synonym: erythrocyte)

You are caring for a client with multiple myeloma. Why would it be important to assess this client for fractures? a) Osteolytic activating factor weakens bones producing fractures. b) Osteopathic tumors destroy bone causing fractures. c) Osteoclasts break down bone cells so pathologic fractures occur. d) Osteosarcomas form producing pathologic fractures.

c) Osteoclasts break down bone cells so pathologic fractures occur. The abnormal plasma cells proliferate in the bone marrow, where they release osteoclast-activating factor. This in turn causes osteoclasts to break down bone cells, resulting in increased blood calcium and pathologic fractures. The plasma cells also form single or multiple osteolytic (bone-destroying) tumors that produce a 'punched-out' or 'honeycombed' appearance in bones such as the spine, ribs, skull, pelvis, femurs, clavicles, and scapulae. Weakened vertebrae lead to compression of the spine accompanied by significant pain. Options A, C, and D are distractors for this question.

A nurse is assessing a client with multiple myeloma. Due to this condition, what will this client be at risk for? chronic liver failure. acute heart failure. pathologic bone fractures. hypoxemia.

pathologic bone fractures. Explanation: Clients with multiple myeloma are at risk for pathologic bone fractures secondary to diffuse osteoporosis and osteolytic lesions. Also, clients are at risk for renal failure secondary to myeloma proteins by causing renal tubular obstruction. Liver failure and heart failure aren't usually sequelae of multiple myeloma. Hypoxemia isn't usually related to multiple myeloma.

A nurse is assessing a client with multiple myeloma. The nurse should keep in mind that clients with multiple myeloma are at risk for:

pathologic bone fractures. Clients with multiple myeloma are at risk for pathologic bone fractures secondary to diffuse osteoporosis and osteolytic lesions. Also, clients are at risk for renal failure secondary to myeloma proteins by causing renal tubular obstruction. Liver failure and heart failure aren't usually sequelae of multiple myeloma. Hypoxemia isn't usually related to multiple myeloma.

Lymphoid

pertaining to lymphocytes

Myeloid

pertaining to nonlymphoid blood cells that differentiate into RBCs, platelets, macrophages, mast cells, and various WBCs

Blast cell

primitive leukocyte

The nursing instructor is discussing disorders of the hematopoietic system with the pre-nursing patho class. What disease would the instructor list with a primary characteristic of erythrocytosis?

Polycythemia vera

The nurse is interacting with a family that has been caring for a patient with cancer for several months. What are the best interventions to assist in relieving caregiver stress in this family? Select all that apply. a) Suggest the family go to church more often. b) Suggest the prescription of anti-anxiety medications. c) Suggest support for household maintenance. d) Allow family members to express feelings. e) Educate the family about medications and side effects.

c) Suggest support for household maintenance., d) Allow family members to express feelings., e) Educate the family about medications and side effects. Explanation: Family members benefit from increased education on what to expect. Allowing family members to express their feelings has also been shown to relieve stress. Supporting the caregiver and family with help in household duties will also help the over-burdened family. Anti-anxiety medications and church attendance have not been shown to reduce caregiver stress.

The nurse is teaching the patient about consolidation. What statement should be included in the teaching plan? a) "Consolidation of the lungs is an expected effect of induction therapy." b) "Consolidation occurs as a side effect of chemotherapy." c) "Consolidation of the lungs is an expected effect of induction therapy." d) "Consolidation therapy is administered to reduce the chance of leukemia recurrence."

d) "Consolidation therapy is administered to reduce the chance of leukemia recurrence." Explanation: Consolidation therapy is administered to eliminate residual leukemia cells that are not clinically detectable and reduce the chance for recurrence. It is also termed post-remission therapy. It is not a side effect of chemotherapy, but the administration of chemotherapy.

Which assessment findings support the client's diagnosis of AML (acute myeloid leukemia)? Select all that apply. a) Bone pain b) Enlarged lymph nodes c) Enlarged heart d) Weakness and fatigue e) Petechiae

a) Bone pain, b) Enlarged lymph nodes, d) Weakness and fatigue, e) Petechiae Explanation: Clients with AML may present with petechiae, enlarged lymph nodes, weakness, fatigue, and bone pain. An enlarged heart is not a typical finding with this disorder.

A patient with multiple myeloma is complaining about pain. What instructions will the nurse give the patient to help to reduce pain during activity? a) Do not lift more than 10 pounds. b) Limit activity to once a day. c) Limit fluids to prevent going to the bathroom. d) Stay in bed as much as possible.

a) Do not lift more than 10 pounds. Explanation: The patient with multiple myeloma needs education about activity instructions such as lifting no more than 10 pounds and using proper body mechanics. Braces may be needed. The patient should have activity and would not be instructed to stay in bed or limit activity as he or she would become very stiff. Limiting fluids would be contraindicated. The patient needs to remain well hydrated.

After chemotherapy for AML, what interventions will best help to prevent renal complications? Select all that apply. a) Increase hydration. b) Administer allopurinol (Zyloprim). c) Encourage exercise. d) Administer potassium therapy. e) Administer rasburicase (Elitek).

a) Increase hydration., b) Administer allopurinol (Zyloprim)., e) Administer rasburicase (Elitek). Explanation: Increased uric acid and phosphorus levels after chemotherapy for AML can lead to renal calculi formation. Increasing hydration and administering allopurinol (a uricosuric) will help to eliminate the uric acid. Elitek is an enzyme that can also decrease uric acid. Administration of potassium is not indicated as levels are elevated after chemotherapy. Exercise is not initially encouraged because the patient could have weakness and cramping during this time.

Which of the following terms refers to a form of white blood cell involved in immune response? a) Lymphocyte b) Spherocyte c) Thrombocyte d) Granulocyte

a) Lymphocyte Explanation: Both B and T lymphocytes respond to exposure to antigens. Granulocytes include basophils, neutrophils, and eosinophils. A spherocyte is a red blood cell without central pallor, seen with hemolysis. A thrombocyte is a platelet.

The term that is used to refer to a primitive cell, capable of self-replication and differentiation, is which of the following? a) Stem cell b) Band cell c) Reticulocyte d) Spherocyte

a) Stem cell Explanation: Stem cells may differentiate into myeloid or lymphoid stem cells. A band cell is a slightly immature neutrophil. A spherocyte is a red blood cell without central pallor. A reticulocyte is a slightly immature red blood cell.

When assessing a female patient with a disorder of the hematopoietic or the lymphatic system, which of the following assessments is most essential? a) Lifestyle assessments, such as exercise routines b) Age and gender c) Menstrual history d) Health history, such as bleeding, fatigue, or fainting

d) Health history, such as bleeding, fatigue, or fainting Explanation: When assessing a patient with a disorder of the hematopoietic or the lymphatic system, it is essential to assess the patient's health history. An assessment of drug history is essential because some antibiotics and cancer drugs contribute to hematopoietic dysfunction. Aspirin and anticoagulants may contribute to bleeding and interfere with clot formation. Because industrial materials, environmental toxins, and household products may affect blood-forming organs, the nurse needs to explore exposure to these agents. Age, gender, menstrual history, or lifestyle assessments, such as exercise routines and habits, do not directly affect the hematopoietic or lymphatic system.

A patient has completed induction therapy and has diarrhea and severe mucositis. What is the appropriate nursing goal? a) Address issues of negative body image. b) Place client in reverse isolation. c) Administer pain medication. d) Maintain nutrition.

d) Maintain nutrition. Explanation: Maintaining nutrition is the most important goal after induction therapy because the patient experiences severe diarrhea and can easily become nutritionally deficient as well as develop fluid and electrolyte imbalance. The patient is most likely not in pain at this point, and this is an intervention not a goal.

The nurse is interacting with a family that has been caring for a pt with cancer for several months. What are the best interventions to assist in relieving caregiver stress in this family? (multiple answers)

educate the fam about meds and side effects, allow fam members to express feelings, and suggest support for household maintenance

A pt with acute myeloid leukemia (AML) had a neutrophil count that persists at less than 100/mm3. What should the nurse cautiously monitor this pt for?

infection

Which cell of hematopoiesis is responsible for the production of red blood cells (RBCs) and platelets? a) Myeloid stem cell b) Monocyte c) Neutrophil d) Lymphoid stem cell

a) Myeloid stem cell Explanation: The myeloid stem cell is responsible not only for all nonlymphoid white blood cells (WBC), but also for the production of RBCs and platelets. Lymphoid cells produce either T or B lymphocytes. A monocyte is large WBC that becomes a macrophage when is leaves the circulation and moves into body tissues. A neutrophil is a fully mature WBC capable of phagocytosis.

The nurse is teaching a patient with acute lymphocytic leukemia (ALL) about therapy. What statements should be included in the plan of care? a) "The goal of therapy is palliation." b) "Side effects are rare with therapy." c) "Treatment is simple and consists of single drug therapy." d) "Intrathecal chemotherapy is used primarily as preventive therapy."

d) "Intrathecal chemotherapy is used primarily as preventive therapy." Explanation: Intrathecal chemotherapy is a key part of the treatment plan to prevent invasion of the central nervous system. The therapy uses multiple drugs, with many side effects. The goal of therapy is remission.

A patient who is being treated for AML has bruises on both legs. What is the nurse's most appropriate action? a) Evaluate the patient's platelet count. b) Keep the patient on bed rest. c) Ask the patient if he has been falling recently. d) Evaluate the patient's INR.

a) Evaluate the patient's platelet count. Explanation: Complications of AML include bleeding. The risk of bleeding correlates with the level and duration of platelet deficiency. Major hemorrhages may develop when the platelet count drops to less than 10,000 x mm3. The bleeding is usually unrelated to falling. Keeping the patient on bed rest will not prevent bleeding when the patient has a low platelet count. Assessment for other areas of bleeding is also a priority intervention.

Which statement indicates the patient understands teaching about induction therapy for leukemia? a) "I know I can never be cured." b) "I will start slowly with medication treatment." c) "I will need to come every week for treatment." d) "I will be in the hospital for several weeks."

d) "I will be in the hospital for several weeks." Explanation: Induction therapy involves high doses of several medications and the patient is usually admitted into the hospital for several weeks. The treatment is started quickly and the goal is to cure or put the patient into remission.

The nurse is caring for a client with multiple myeloma. Why would it be important to assess this client for fractures?

Osteoclasts break down bone cells so pathologic fractures occur. The abnormal plasma cells proliferate in the bone marrow, where they release osteoclast-activating factor. This, in turn, causes osteoclasts to break down bone cells, resulting in increased blood calcium and pathologic fractures. The plasma cells also form single or multiple osteolytic (bone-destroying) tumors that produce a "punched-out" or "honeycombed" appearance in bones such as the spine, ribs, skull, pelvis, femurs, clavicles, and scapulae. Weakened vertebrae lead to compression of the spine accompanied by significant pain. The other options are distractors for this question.

Which of the following statements best describes the function of stem cells in the bone marrow? a) They defend against bacterial infection. b) They produce antibodies against foreign antigens. c) They produce all blood cells. d) They are active against hypersensitivity reactions.

c) They produce all blood cells. Explanation: All blood cells are produced from undifferentiated precursors called pluripotential stem cells in the bone marrow. Other cells produced from the pluripotential stem cells help defend against bacterial infection, produce antibodies against foreign antigens, and are active against hypersensitivity reactions.

For a patient with Hodgkin disease, who is at a risk for an ineffective airway clearance and an impaired gas exchange, the nurse places the patient in a high Fowler's position to do which of the following? a) Increase the lung expansion. b) Anticipate the need for the airway management. c) Reduce the deficits in the blood oxygen level. d) Detect compromised ventilation.

a) Increase the lung expansion. Explanation: For a patient with Hodgkin disease who is at a risk for an ineffective airway clearance and an impaired gas exchange, the nurse keeps the neck in midline and places the patient in a high Fowler's position if respiratory distress develops. Avoiding unnecessary pressure on the trachea and positioning for an increased lung expansion improve the air exchange. The nurse administers oxygen as per the physician's orders to reduce the deficits in the blood oxygen level. The nurse assesses the respiratory status in each shift to detect compromised ventilation. The nurse places an endotracheal tube, a laryngoscope, and a bag-valve mask at the bedside for intubation if the need for the airway management arises.

A patient with AML is having aggressive chemotherapy to attempt to achieve remission. The patient is aware that hospitalization will be necessary for several weeks. What type of therapy will the nurse explain that the patient will receive?

Induction therapy Despite advances in understanding of the biology of AML, substantive advances in treatment response rates and survival rates have not occurred for decades, with the exception of advances made in treating APL (see later discussion). Even for patients with subtypes that have not benefited from advances in treatment, cure is still possible. The overall objective of treatment is to achieve complete remission, in which there is no evidence of residual leukemia in the bone marrow. Attempts are made to achieve remission by the aggressive administration of chemotherapy, called induction therapy, which usually requires hospitalization for several weeks.


Conjuntos de estudio relacionados

General Trivia Questions (Version 2)

View Set

Unit 15 & 17 - Care of Family Ch. 22

View Set

IP ADDRESSING -PART 2​ Lecture 16

View Set

Farm Business Management Midterm

View Set

US History Chapters 21.3, 22.2, and 22.4

View Set

A&P Chapter 17- Endocrine System

View Set

Human Bio - Chapter 11 Study Guide

View Set